You are on page 1of 481
ELEMENTS OF THE DIFFERENTIAL AND INTEGRAL CALCULUS ® (REVISED EDITION) | BY WILLIAM ANTHONY GRANVILLE, Pu.D., LL.D. PRESIDENT OF GETTYSBURG COLLEGE WITH THE EDITORIAL COUPERATION OF PERCEY F. SMITH, Pu.D. PROFESSOR OF MATHEMATICS IN-THE SHEFFIELD SCIENTIFIC SCHOOL YALE UNIVERSITY GINN AND COMPANY BOSTON + NEW YORK + CHICAGO - LONDON ATLANTA + DALLAS + COLUMBUS - SAN FRANCISCO corynn r, 1904, 1911, BY WinitAM ANTHONY GRANVILLE AND Percy F. SMITH ALL RIGHTS RESERVED ont Tbe Atheneum Press GINN AND COMPANY - PRO- PRIETORS - BOSTON U.S.A. PREFACE That teachers and students of the Calewlus have shown such a gen- erous appreciation of Granville’s “Elements of the Differential and Integral Calculus” has been very gratifying to the author. In the last few years considerable progress has been made in the teaching of the elements of the Calculus, and in this revised edition of Granville’s “Calculus” the latest and best methods are exhibited, — methods that: have stood the test of actual classroom work. Those features of the first edition which contributed so much to its usefulness and popu- larity have been retained. The introductory matter has been cut down somewhat in order to get down to the real business of the Calculus sooner, As this is designed essentially for a drill book, the pedagogic principle that each result should be made intuitionally as well as analytically evident to the student has been kept constantly in mind. The object is not to teach the student to rely on his intuition, but, in some cases, to use this faculty in advance of analytical investigation. Graphical illustration has been drawn on very liberally. ‘This Calculus is based on the method of limits and is divided into two main parts, — Differential Calculus and Integral Calculus. As special features, attention may be called to the effort to make per- fectly clear the nature and extent of each new theorem, the large number of carefully graded exercises, and the summarizing into working rules of the methods of solving problems. In the Integral Calculus the notion of integration over a plane area has been much enlarged upon, and integration as the limit of a summation is con- stantly emphasized. The existence of the limit ¢ has been assumed and its approximate value calculated from its graph. A large num- ber of new examples have been added, both with and without answers. At the end of almost every chapter will be found a col- lection of miscellaneous examples. Among the new topics added are approximate integration, trapezoidal rule, parabolic rule, orthogonal : vi PREFACE trajectories, centers of area and volume, pressure of liquids, work done, etc. Simple practical problems have been added throughout; problems that illustrate the theory and at the same time are of interest to the student. These problems do not presuppose an ex- tended knowledge in any particular branch of science, but are based on knowledge that all students of the Calculus are supposed to have in common. ‘The author has tried to write a textbook that is thoroughly modern and teachable, and the capacity and needs of the student pursuing a first course in the Calculus have been kept constantly in mind. The book contains more material than is necessary for the usual course of one hundred lessons given in our colleges and engineering schools ; but this gives teachers an opportunity to choose such subjects as best suit the needs of their classes. It is believed that the volume con- tains all topics from which a selection naturally would be made in preparing students either for elementary work in applied science or for more advanced work in pure mathematics. WILLIAM A. GRANVILLE Gurrvsnone Connnce Gettysburg, Pa. CONTENTS DIFFERENTIAL CALCULUS CHAPTER I COLLECTION OF FORMULAS SECTION 1. Formulas from Algebra, Trigonometry, and Analytic Geometry 2. Greek alphabet 8. Rules for signs in the four quadrants 4. Natural values of the trigonometric functions 5. Tables of logarithms CHAPTER IL VARIABLES AND FUNCTIONS 6. Variables and constants : . . : . . + . 7. Interval of a variable. : . . : : : . . 8. Continuous variation 9. Functions 10. Independent and dependent variables 11. Notation of functions 12. Values of the independent variable for which a function is defined CHAPTER IIT THEORY OF LIMITS 13. Limitofavariable =... 14. Division by zero excluded 15. Infinitesimals . . . . * . . . 16. The concept of infinity _ 17. Limiting value of afunction. =... we 18. Continuous and discontinuous functions 19. Continuity and discontinuity of functions illustrated by their graphs 20. Fundamental theorems on limits . aoe 21. Special limiting values . 22. The limit of as 28. The number e 5 24. Expressions assuming the form =... 5s vii race aR eee warraaa abl 12 13 13 = ney 16 18 20 21 22 23, ay viii CONTENTS CHAPTER IV DIFFERENTIATION SECTION 25. Introduction . 26, Increments 27. Comparison of increments 98. Derivative of a function of one variable 29. Symbols for derivatives . 30. Differentiable functions . 31. General rule for differentiation 82. Applications of the derivative to Geometry CHAPTER V PAGE 25 25 26 27 28 29 29 BL RULES FOR DIFFERENTIATING STANDARD ELEMENTARY FORMS 33. Importance of General Rule . 84. Differentiation of a constant . 85. Differentiation of a variable with respect to itself. 36. Differentiation of a sum 87. Differentiation of the product of a constant and a function 38. Differentiation of the product of two functions 39. Differentiation of the product of any finite number of fumetions 40. Differentiation of a function with a constant exponent . 41. Differentiation of a quotient . 42. Differentiation of a function of a function 43. Differentiation of inverse functions 44. Differentiation of a logarithm 5 45. Differentiation of the simple exponential function . 46. Differentiation of the general exponential function 47. Logarithmic differentiation 48. Differentiation of sin» . 49. Differentiation of cos» . 50. Differentiation of tanv . 51. Differentiation of cotv . 2. Differentiation of seev . . Differentiation of esev . Differentiation of versv. 5. . Differentiation of are sin» . Differentiation of are cos » . Differentiation of are tan o . Differentiation of are cot » . Differentiation of arc see » . Differentiation of are ese » 34 36 37 387 37 38 38 39 40 44 45 46 48 49 50 54 5D 56 56 56 BT 57 61 62 62 63 63 64 | CONTENTS ix \ SECTION PAGE | G1. Differentiation of areversv 2 see OB | 62. Implicit functions . S . i. = eS . oe 68. Differentiation of implicit functions. 9. 0... ss OD CHAPTER VI SIMPLE APPLICATIONS OF THE DERIVATIVE 64. Direction of a curve. 73 65. Equations of tangent and normal, lengths of subtangent and subnormal. Rectangular cobrdinates. 5. sete TB 66. Parametric equations of acurve . 79 i 67. Angle between the radius vector drawn on point Go ours oa as | tangent to the curve at that point 7 . rn: | 68. Lengths of polar subtangent and polar subnormal =... 86 i 69. Solution of equations having multiple roots. oe 70. Applications of the derivative in mechanics. Velocity. . | | 90 Z. Componcutvelocitics © = 72. Acceleration a h—hlrar—Ss=s=SsaC 78. Component accelerations =». sw wee 8 CHAPTER VIL SUCCESSIVE DIFFERENTIATION \ 74, Definition of successive derivatives 2... wes | 75. Notation. err——ssss i 76. The nth derivative | ot TT. Leibnita’s formula for the nth derivative of a product. =. =. 98 78. Successive differentiation of implicit functions . =. =... 100 CHAPTER VIIT MAXIMA AND MINIMA. POINTS OF INFLE! TION. CURVE TRACING 79. Introduction —. a... i 80. Increasing and decreasing functions . 106 81. Tests for determining when a function is increasing and when de- creasing =. oo le 82. Maximum and minimum values of function | » 109 i 88. First method for examining a function for maximum and minimum values. 0. = ll | 84. Second method for examining a function for maximum and minimum values. 12 ..LDrt~”C”—OC—COdC;SOOSCORSCSC«SCiSCdsCdsSCSsS¥SCéséSKsé flection. Be 185 i SGcCuvetuacme Ce : CONTENTS CHAPTER IX DIFFERENTIALS SECTION 87. 88. 89. 90. 91. 92. 93. 94. 95. 96. 97. 98. 99. 100. 101. 102. 103. 104. 105. 106. 107. Introduction Definitions Infinitesimals : Derivative of the aro in restangular cobrdinates Derivative of the are in polar codrdinates . Formulas for finding the differentials of functions Successive differentials . . . . CHAPTER X RATES ‘The derivative considered as the ratio of two rates CHAPTER XI CHANGE OF VARIABLE Interchange of dependent and independent variables . Change of the dependent variable Change of the independent variable . Simultaneous change of both independent and dependent variables CHAPTER XII CURVATURE, RADIUS OF CURVATURE Curvature . 5 Curvature of a circle . Curvature at a point . Formilas for curvature Radius of curvature . : : : - Circle of curvature. 5 a _ a CHAPTER XIII ‘THEOREM OF MEAN VALUE. INDETERMINATE FORMS Rolle’s Theorem oo inecrheona cP Moak vale 8 oN ce The Extended Theorem of Mean Value... PAGE 131 181 182 134 135 187 139 141 148 149 150 152 155, 155 156 156 159 161 164 165 166 CONTENTS SECTION 108. Maxima and minima treated a 7 109. Indeterminate forms . : 110. Evaluation of a function taking ... 111. Evaluation of the indeterminate form : oo 112. Evaluation of the indeterminate form... we 113. Evaluation of the indeterminate form 0-0 114. Evaluation of the indeterminate form « — 0 7 115. Evaluation of the indeterminate forms 0°,1°,0® . . CHAPTER XIV CIRCLE OF CURVATURE. CENTER OF CURVATURE 116. Circle of curvature. Center of curvature 117. Second method for finding center of curvature . 118. Center of curvature, the limiting position of the intersection of nor- mals at neighboring points . 119. Evolutes . 120. Properties of the evolute : 121. Involutes and their mechanical construction CHAPTER XV PARTIAL DIFFERENTIATION 122. Continuous functions of two or more independent variables 128. Partial derivatives G6 124, Partial derivatives interpreted geometrically |. 125. Total derivatives 126. Total differentials so 197. Differentiation of implicit functions. =. se 128. Successive partial derivatives 2. 0... 129. Order of differentiation immaterial CHAPTER XVI ENVELOPES 130. Family of curves. Variable parameter... 181. Envelope of a family of curves depending on one parameter : The evolute'of a given curve considered as the envelope of its normals . Two parameters connected by one equation of condition xi Page 167 170 170 171 174 174 175 176 178 180 181 182 186 187 190 191 192 194 197 198 202 208 205 205 208 209 xii CONTENTS CHAPTER XVII SECTION ee PAGE, eae tutroducom le 135. Infinite series. ee, OB 186. Existence of a limit . eg 187. Fundamental test for eonvergence | 5 sss RB 138. Comparison test for convergence =... ws 189. Cauchy's ratio test for convergence... www 8. 40 Alternating cores rl Cl 141. Absolute convergence =. ss we eee 200 dea Powerserieg 2 ts—ri—“(<—er”ssts=aC=s=CzdCl CHAPTER XVIII EXPANSION OF FUNCTIONS 143. Introduction eo 14. Taylor's Theorem and Taylor's Series =. | sss. 298 145. Maclaurin’s Theorem and Maclaurin’s Series . =... . 280 146. Computation by series. so eee 147. Approximate formulas derived from series. Interpolation. | | 987 148. Taylor’s Theorem for functions of two or more variables... =. 240 149. Maxima and minima of functions of two independent variables. 243 CHAPTER XIX ASYMPTOTES, SINGULAR POINTS 150. Reetilinear asymptotes ke 151. Asymptotes found by method of limiting intercepts. |. 240 152. Method of determining asymptotes to algebraic curves... 250 153. Asymptotes in polar codrdinates =. sw wwe 154. Singular points... 5 255 155. Determination of the tangent to an algebraic curve at a given point Dy inepection os 156. Nodes. we ee 888 157. Cusps we ee 158. Conjugate or isolated points «=» | | ss est 260 159. Transcendental singularities . 2... +. ey 260 CHAPTER XX APPLICATIONS TO GEOMETRY OF SPACE 160. ‘Tangent line and normal plane to a skew curve whose equations are Given inperemeticform S20 16l Tangent plenstaeourtece CONTENTS seotiox 162. Normal line to a surface : 168. Another form of the equations of the tangent line to a skew enrve 164. Another form of the equation of the normal plane to a skew curve CHAPTER XXI CURVES FOR REFERENCE INTEGRAL CALCULUS CHAPTER XXII INTEGRATION. RUL FOR INTEGRATING STANDARD ELE) FORMS 165. Integration oo. 166. Constant of integration, Indefinite integral 167. Rules for integrating standard elementary forms 168. Trigonometrie differentials 169. Integration of expressions containing Va? — 2? or VE + a bya trigo- nometric substitution . CHAPTER XXIII CONSTANT OF INTEGRATION 170. Determination of the constant of integration by means of initial con- ditions 171. Geometrical signification of the constant of integration 172. Physical signification of the constant of integration . CHAPTER XXIV THE DEFINITE INTEGRAL 178. Differential of an area 174. The definite integral . 175. Calculation of a definite ine 176. Calculation of areas . : 177. Geometrical representation of an antegral - 178. Mean value of $(2) . 179. Interchange of limits 180. Decomposition of the interval 181. The definite integral a function of its limits 182. Infinite limits 188. When y = $(2) is discontinuous xiii PAGE 266 268, 269 ENTARY, 279 281 282 298 304 307 307 309 314 34 316 318 319 320 320 320 321 321 822 xiv sEori0N 184. 185. 186. 187. 188. 189. 190. 191. 192. 198. 194. 195. 196. 197. 198. 199. 200. 201. 202. CONTENTS CHAPTER XXV INTEGRATION OF RATIONAL FRACTIONS Introduction 53 Col Case II Case IIT Case IV CHAPTER XXVI INTEGRATION BY SUBSTITUTION OF A NEW VARIABLE. RATIONALIZATION Introduction Differentials containing fruetional powers of z only Differentials containing fractional powers of a + bx only Change in limits corresponding to change in variable Differentials containing no radical except Va + bx + 2? Differentials containing no radical except Va + be — 2? Binomial differentials : : Conditions of integrability of binomial differentials . ‘Transformation of trigonometric differentials Miscellaneous substitutions CHAPTER XXVII INTEGRATION BY PARTS. REDUCTION FORMULAS Formula for integration by parts . Reduction formulas for binomial differentials Reduction formulas for trigonometric differentials To find fe sin nede and fe cos nede CHAPTER XXVIII INTEGRATION A PROCESS OF SUMMATION }. Introduction . . ‘The fundamental theorem of Integrel Celculus . . Analytical proof of the Fundamental Theorem . . Areas of plane curves. Rectangular codrdinates . Area when curve is given in parametric form . Areas of plane curves. Polar codrdinates . . Length of a curve poo . Lengths of plane curves. Rectangular codrdinates . Lengths of plane curves. Polar codrdinates PAGE 825 825 327 329 331 335, 335 336 336 338 338 340 341 843, 345 347 350 356 359 361 361 364 365, 368 370 372 373, 875 7 CONTENTS SECTION 212. 213. 214. 215. 216. 217. 218. 219. 220. 221. 222. 224. 225. 226. 227. 228, 229. 230. 231. 232. Volumes of solids of revolution . Areas of surfaces of revolution . Miscellaneous applications. =... we CHAPTER XXIX SUCCESSIVE AND PARTIAL INTEGRATION Successive integration Partial integration oe Definite double integrai. Geometric interpretation Value of a definite double integral over a region Plane area as a definite double integral, Rectangular codrdinates Plane area as a definite double integral. Polar codrdinates ‘Moment of area. Center of area Moment of inerti: Plane a areas: Polar moment of inertia, Rectangular eodrdinates Polar moment of ine General method for finding the areas of surfaces Volumes found by triple integration . Polar coérdinates CHAPTER XXX ORDINARY DIFFERENTIAL EQUATIONS Differential equations. Order and degree . Solutions of differential equations Verifications of solutions ; Differential equations of the first order and of the first degree . Differential equations of the nth order and of the first degree CHAPTER XXXI xv AGE 3877 381 385, 393, 395, 396 400 402 406 408 408 410 410 411 413 417 421 422 423 424 432 INTEGRAPH. APPROXIMATE INTEGRATION, TABLE OF INTEGRALS 238. 234. 235. 236. 237. 238. 239. 240. 241. ‘Mechanical integration Integral curves . ‘The integraph Polar planimeter A Area swept over by a line . Approximate integration Trapezoidal rule 5 Simpson’s rule (parabolic rule) . Integrals for reference INDEX . . . 443 443, 445 446 446 448 448 449 451 461 DIFFERENTIAL CALCULUS CHAPTER I COLLECTION OF FORMULAS 1, Formulas for reference. For the convenience of the student we give the following list of elementary formulas from Algebra, Geome- try, Trigonometry, and Analytic Geometry. 1, Binomial Theorem (n being a positive integer) : (a4 De = an § nanrb 4 MEX D an -aye 4 MOTOR gp ROAD ABD MATF®) ger ttor a gee, a 808 aes 2 ni=[n=1-2-8-4...(n—1)n : a 3. In the quadratic equation az? + be +¢ = 0, when b? — 4ac > 0, the roots are real and unequal ; when 0 — 4ac = 0, the roots are real and equal ; when 8? — 4ac <0, the roots are imaginary. 4, When a quadratic equation is reduced to the form 2° + px + =0, p= sum of roots with sign changed, and q = product of roots, 5. In an arithmetical series, at(@—1)d; n ae Zt =52utm—1al. 6. Ina geometrical series, rl—a_a(m—1) 1=am-1 = r-1 or—1 1: log. ab = log a + log. 10. log Va = 2 log a. 18. log 2 =— log a. 8. log # = loga—logb. 11. log 1 = 0. 14. Circumference of cirele=2mr. 9. log a" = nlog a. 12. logaa 15. Area of circle = mr’, * In formulas 14-25, r denotes radius, a altitude, B area of base, and s slant height. a to DIFFERENTIAL CALCULUS 16. Volume of prism = Ba. 17. Volume of pyramid = 3 Ba. 18. Volume of right circular cylinder = ma, 19. Lateral surface of right circular cylinder = 20. Total surface of right circular eylinder = 2r(r + a). 21, Volume of right circular cone = } ma, 22, Lateral surface of right circular cone = rs, 23, Total surface of right circular cone = wr(r + 8). 24. Volume of sphere = $ ar’. 25, Surface of sphere = 4 mr’, ra. 1 1 1 26. sinz = ——; cost = ——; tang =—_. wor seca oot ine cos 27. tang 3 cota = 2 cost sing 1 28. sin?z + cos*x =1; 14 tanta = see®a; 1+ cot?x = cscta, 29. sine = cos (Z — s 31. sin(@@ + y) =sinz cosy + cosasiny. 32, sin (e — y) = sin x cosy — cose siny. cons = sin (Z = 5 a 33. cos(t + y) = cose cosy ¥ sinzsiny. tanz = oot (= ). 2 34. tan(z + y) = me titony | : : T—tanetany 30, sin (wz) = sina; cos (# — 2) =— cos; 35, tan(e— y= 2B2— tan tan (w— 2) =— tang. T+ tanztany 36, sin2e = 2Qsinz cos; cos2e= costa — sintz; tan22—= Re, 1— tan?x 37. sing = 2sin¥ cos’; cosa = cost — sin? tans as 38. costa = 4 + }cos2z; sin?e = }— } cos2e, 39. 14 cosa = 2 cost; 1— cosa = asin? 40. sin 5 = eaf—O*) cost = 44 pees tang = 41. FEO AE 42. sin — sin y = 2 cos] ( + y)sin} (a — ). 43. cosx + cosy = 2 c0s}(x + y) cos} (e— y). 44. cosz — cosy =— 2sin} (x + y)sin} (e—y). i — cose 1+ cosa Gi Law of Sines. 46, a? =U + c? — 2be cos A; Law of Cosines, 41. d=V(@,— 2," + WH, — Vo} distance between points (2, 1 and (tq) ¥s)+ An + Buy +0 48. d= te; £VA +B distance from line Az + By + C = 0 to (t,, %)- COLLECTION OF FORMULAS 3 49, 2-0 tt yo Mth 2 2 50. e=a)+2', y= yt Y's transforming to new origin (x, ¥). BL. 2 = 2 cosd— sind, the angle 6 with old. 52. «=p cosé, y =p sind; transforming from rectangular to polar cobrdinates, ; codrdinates of middle point. =a’ sind + f cos; transforming to new axes making 53. p=Vae+y, O=are tan z ; transforming from polar to rectangular codrdinates. 54, Different forms of equation of a straight line: fa) = =e = ‘two-point form; () £441, intercept form; (©) y— y= me — 2), slope-point form; (@) v= me + b, slope-intercept form ; (e) 200s @ + y sin a= p, normal form ; () Ax + By + C= 0, general form, 55. tand = itm? angle between two lines whose slopes are m, and my, ‘m, =m, when lines are parallel, and my=— x when lines are perpendicular. 56. (@ — a)? + (y — 6)? = °, equation of circle with center (a, 8) and radius r, 2. Greek alphabet. Letters Names Letters Names’ Letters Names Aa Alpha dk Tota Pp Rho BB Beta K « Kappa = os Sigma T y Gamma A 2X Lambda Tr Tau A & Delta Mp Mu ev. Upsilon. Ee Epsilon. Nv Nu ® ¢~ Phi ZE Zeta BE Xi KX xy Chi H Eta © 0 — Omicron Vy Psi @ 6 Theta iz bi Q o Omega 3. Rules for signs of the trigonometric functions. Quadrant | Sin Cos ‘Tan Cot See se First + + + + Second | . . = = ao . ‘i Thi - - + + = a DIFFERENTIAL CALCULUS 4, Natural values of the trigonometric functions. Angle in Angle in Smead pet ae Cos tan cot See se 0 ° 0 1 0 » i © 7 90° 1 0 * 0 © al 7 180° 0 ae 0 o a | o oe 270° — 0 - 0 ~ fe 2 Qa 360° 0 a 0 oO . o ‘Angle in | Angle in : - : ceca! pee) cos ‘tan cot See xe 0 oe 0 7 0 o 2 2 x 1 v3 v3 vi 2Vv3 B a = = 3 — 2 6 2 2 3 3 m= va va = 45° v2 v2 1 1 2 2 4 2 2 @ 2 7 vB 1 v3 2v3 e 60° ~ 5 8 aly eney 8 2 2 ~ 3 : 3 = 2g 90° 7 0 co oO o 1 Anglo in | Angle in : - Radians Degrees: bee Goi ee Sed 0000 ° .0000 | 1.0000 | .0000 2 90° | 1.6708 0175, i 0175 +9998 0175 57,290 Bm 1,5533 0349 2 0849 | .9004 | 0349 | 28.636 | 88° | 1.5859 0524 a 0523 | .9086 | 0524 | 19.081 | 87° | 1.5184 0698 4 0698 9976 0699 14,300 86° 1.5010 0873 & 0872 | 9062 | .0875 | 11.480 | 85° | 1.4885 «1745 10° 1736 | 9848 | 1763 | 5.671 | 80° | 1.8968 2618 15° 2588 9659 2679 ‘8.782 15 1.3090 3491 20° 8420, 9897 3640 2.747 70° 1.2217 4368 26° 4226 | .9063 | 4608 | 2.145 | 65° | 1.1345, 5236 30° 5000, 8660 174 1.732, 60° 1.0472 6109 36° 5736 | .8192 | .7002 | 1.428 | 55° 9509 6981 40° 6428 | .7660 | .9301 | 1.192 | 50° 8727 «7854 45° 7071 7071 1.0000 1,000, 45° 7854, : = ‘Angie in | Angle in os sin Cot aon oe | ace COLLECTION OF FORMULAS 5. Logarithms of numbers and trigonometric functions. Taste or Manvissas or THE Common Locaritums or Numbers No. O _ 2 3 4 5 6 7 8 9 1 |] 0000 | 0414 | o792 | 1139 | 1461 || 1761 | 2041 2553 | 2788 2 |] 3010 | 8222 | 8424 | 3617 | 3802 || 3979 | 4150 4472 | 4024 3 |] 4771 | 4914 | 5051 | 5185 | 5816 || 5441 | 5563 5798 | 5911 4 || 6021 | 6128 | 232 | 6335 | 6485 |] 6582 | 628 6812 | 6902 5 |} 990 | 7076 | 7160 | 7243 | 7324 || 7404 | 7482 7634 | 7709 6 || 7782 | 7858 | 7924 | 7993 | 8062 || 8120 | 8195 9325 | 8388 7 || 8451 | 8518 | 8573 | 8638 | 8692 || 8751 | 8808 8921 | 8976 8 || 9031 | 9085 | 9138 | 9191 | 9243 || 9204 | 9345 9445 | 9494 9 || 9542 | 9590 | 9088 | 9685 | 9731 || 9777 | 9823 gg12 | 9956 10 || 0000 “0043 | “0086 | "0128 | 0170 || 0212 | “0253 03384 | 087d 11 |) nit | 0458 | “0492 | “0581 |~0569 || 0607 | 0645 0719 | 0755 12 |] 0792 | 0828 | 0864 | 0899 | 0984 || 0969 | 1004 1072 | 1106 13 || 1139 | 1178 | 1206 | 1239 | 1271 || 1808 | 1885 | 1367 | 1399 | 1430 14 || 1461 | 1492 | 1528 | 1553 | 1584 |} 1614 | 1644 | 1673 | 1703 | 1732 15 || 1761 | 1790 | 1818 | 1847 | 1875 || 1903 | 1981 | 1959 | 1987 | 2014 16 |} 2041 | 2068 | 2095 | 2122 | 2148 || 2175 | 2201 | 2227 | 2253 | 2270 17 || 2804 | 2830 | 2355 | 2880 | 2405 |} 2430 | 2455 | 2480 | 2504 | 2529 18 |] 2558 | 2577 | 2601 | 2625 | 2648 || 2672 | 2695 | 2718 | 2742 | 2765 19 || 2788 | 2810 | 2888 | 2856 | 2878 || 2000 | 2928 | 2945 | 2067 | 2080 Taste or Logarrrams oF THE TRIGONOMETRIC FuN ONS. Angle in | Angle in Angiein [Angletn| ogsin | -togeos | togtan | togeot 0000 oe 0.000 oe eee 90° wos’ |e 9.0000 | s.2i9 | 1.7581 | 89° 0349, - 9.9997 5431 1.4569 ‘88° 10524 | 3° 7104 | 1.2806 | 87° soeos |e seus | 1.1554 | 86° 0873 |e 8.9120 | 1.0580 | 85° 745 | 10 9.2468 | 0.7587 | 80° -2618 | 15° 9.4281 0.5719 TS ssior | 20° 9.5611 | 0.4380 | 70° 4363 25° 9.6687 0.3813 65° 5286 | 30° o.764 | 0.93886 | 60° | 1, 6109 | 35° 9.8152 | 0.1548 | 55° | 0.0509 6981 40? 9.9238 50° 0.8727 -Ta54 | 45° 9.8495 | 0.0000 | 45° | 0.7854 | vegece | tpsin | me.ot | sort [Arete | ante CHAPTER II VARIABLES AND FUNCTIONS 6. Variables and constants, A variable is a quantity to which an unlimited number of values can be assigned. Variables are denoted by the later letters of the alphabet. Thus, in the equation of a straight line, 244-1, ats x and y may be considered as the variable codrdinates of a point: moving along the line. A quantity whose value remains unchanged is called a constant. Numerical or absolute constants retain the same values in all prob- lems, as 2, 5, V7, 7, ete. Arbitrary constants, or parameters, are constants to which any one of an unlimited set of numerical values may be assigned, and they are supposed to have these assigned values throughout the inves- tigation. They are usually denoted by the earlier letters of the alphabet. Thus, for every pair of values arbitrarily assigned to a and 8, the equation e+ $1 represents some particular straight line. 7. Interval of a variable. Very often we confine ourselves to a portion only of the number system. For example, we may restrict our variable so that it shall take on only such values as lie between a and 0, where a and } may be included, or either or both excluded. We shall employ the symbol [a, 6], a being less than 4, to represent the numbers a, }, and all the numbers between them, unless otherwise stated. This symbol [a, 0] is read the interval from a to b. 8. Continuous variation. A variable « is said to vary continuously through an interval [a, 6], when « starts with the value a and increases until it takes on the value 4 in such a manner as to assume the value 6 VARIABLES AND FUNCTIONS 7 of every number between a and 6 in the order of their magnitudes. This may be illustrated geometrically as follows: 2 x b 6 Ta ce B ‘The origin being at O, lay off on the straight line the points 4 and B corresponding to the numbers a and 6. Also let the point P corre- spond to a particular value of the variable 2. Evidently the interval [a, 8] is represented by the segment 4B. Now as z varies continuously from a to 6 inclusive, ie. through the interval [a, 6], the point P gen- erates the segment 4B. 9. Functions. When two variables are so related that the value of the first variable depends on the value of the second variable, then the first variable is said to be a function of the second variable. Nearly all scientific problems deal with quantities and relations of this sort, and in the experiences of everyday life we are con- tinually meeting conditions illustrating the dependence of one quan- tity on another. For instance, the weight a man is able to lift depends on his strength, other things being equal. Similarly, the distance a boy can run may be considered as depending on the time. Or, we may say that the area of a square is a function of the length of a side, and the volume of a sphere is a function of its diameter. 10. Independent and dependent variables. The second variable, to which values may be assigned at pleasure within limits depending on the particular problem, is called the independent variable, or argument ; and the first variable, whose value is determined as soon as the value of the independent variable is fixed, is called the dependent variable, or funetion. Frequently, when we are considering two related variables, it is in our power to fix upon whichever we please as the independent variable ; but having once made the choice, no change of independent variable is allowed without certain precautions and transformations. One quantity (the dependent variable) may be a function of two or more other quantities (the independent variables, or arguments). For example, the cost of cloth is a function of both the quality and quantity ; the area of a triangle is a function of the base and altitude; the volume of a rectangular parallelepiped is a function of its three dimensions, 8 DIFFERENTIAL CALCULUS. 11. Notation of functions. The symbol f(z) is used to denote a function of x, and is read f of x. In order to distinguish between different functions, the prefixed letter is changed, as F(x), $(#), f'(a), ete. During any investigation the same functional symbol always indi- cates the same law of dependence of the function upon the variable. In the simpler cases this law takes the form of a series of analytical operations upon that variable. Hence, in such a case, the same func- tional symbol will indicate the same operations or series of operations, even though applied to different quantities. Thus, if f@=2— 90414, then Also FO+1)=64)- 9641) +14 = 7546, fO)=0—9-0414=14, P-D=C1- 9H + 14 = 34, f3)=8—9-8414=—4, fD=T—9-T+14 = 0, ete. Similarly, $(2, y) denotes a function of z and y, and is read $ of and y. lf o@ya=sin@t+y), then (a, b)=sin(a +), and $(§-0)=sin 5 =1, Again, if Fe ye) =204+8y-122, then F(m, —m, m)= 2m —8m—12 m =—18 m, and FQ, 2%, 1) =2°843-2—12-1=0. Evidently this system of notation may be extended indefinitely. 12. Values of the independent variable for which a function is defined. Consider the functions w@—2xr+5, sing, are tang of the independent variable 2 Denoting the dependent variable in each case by y, we may write y=e—2Qat5, y=sing, y=are tana VARIABLES AND FUNCTIONS 9 In each case y (the value of the function) is known, or, as we say, defined, for all values of 2. This is not by any means true of all functions, as the following examples illustrating the more common exceptions will show. Oy i Here the value of y (i.e. the function) is defined for all values of x except 2=, When 2=6 the divisor becomes zero and the value of y cannot be computed from (1).* Any value might be assigned to the function for this value of the argument. Q y=va. In this case the function is defined only for positive values of 2. Negative values, of x give imaginary values for y, and these must be excluded here, where we are confining ourselves to real numbers only. (8) y=log,e. a>0 Here y is defined only for positive values of z. For negative values of x this function does not exist (see § 19). a (4) y=aresina, y =are cosa. Since sines and cosines cannot become greater than +1 nor less than —1, it follows that the above functions are defined for all values of ranging from —1 to + 1 inclusive, but for no other values. EXAMPLES 1. Given f(2) = 2* — 102? + 812 — 80; show that F(0) =— 380, SW) =y — 10? + 81y —30, FQ) =0, F(a) = a — 10a + 81a — 30, FQ=fH), S (yz) = P28 — 102? + 81 yz — 80, FU)>S(— 8), S@— 2) = 29 — 1622 + 832 — 140, S(-1)=- 6F(6). » . It f(a) = 29 — 8x + 2, find £(0), £1), F(— 1), F(— 4), FAY)- 8 1022 + 81a — 30, and (x) = x4 — 55a? — 2102 — 216, show that (—2), F@)=4(— 8), FG) = o(— 4), FO) + (0) + 246 =0. . If F(a) = 2, find F(0), F(— 8), FQ), F(-2). ad op . Given F(z) = 2 (e — 1) ( + 6) (2 — 3) (@ + 9); show that F()= FQ) =F(-0)=FQ)=F(-) =0. * See § 14, p12. 10 = - 10. 11. 12. DIFFERENTIAL CALCULUS =1 my : TE Fm) = Fz show that F(mry) =F (Mg) _ Mm — My L+F(m)F (mq) 1+ mymy . If p(z) =a, show that ¢ (y)-@ (2) =o +2)- 1= 2, show that T¥e oe tom =o (224). Given ¢ (2) = log: . If f(g) = cos g, show that F(8)=S(— 9) =—-S(@ — 9) =F (FF 4). If F(6) = tan 8, show that Fe6) = 27 (2) 1=TFOF Given y (x) = 2 4 2™ 41; show that ¥0)=38, ¥O=1, ¥@=¥(-4%). It f@)= =e o find f(V3). Ans. —.0204, CHAPTER III THEORY OF LIMITS 13, Limit of a variable. If a variable v takes on successively a seriés of values that approach nearer and nearer to a constant value J in such a manner that |» — J|* becomes and remains less than any assigned arbi- trarily small positive quantity, then v is said to approach the limit 1, ov to converge to the limit l. Symbolically this is written limit v= 1, or, v=. The following familiar examples illustrate what is meant: (1) As the number of sides of a regular inscribed polygon is indefi- nitely increased, the limit of the area of the polygon is the area of the circle. In this case the variable is always less than its limit. (2) Similarly, the limit of the area of the circumscribed polygon is also the area of the circle, but now the variable is always greater than tts Limit. (3) Consider the series Hoa (4) Tease The sum of any even number (27) of the first terms of this series is 4 tt al al: 1 Syal—5+q—gt tg ot 2 1 (B) Su= 53 By 6, p.1 Similarly, the sum of any odd number (2”-+1) of the first terms of the series is 111 1061 Smu=l—gt+g—gts— gat ge oT gen Al ©) gay eo By 6, p.1 # To be read the numerical value of the difference between v and l. 12 DIFFERENTIAL CALCULUS Writing (B) and (@) in the forms 2 1 F- Seager Sen Gay limit (2 _ limit 1 we have : " G-s.)=,) reaeh end limit (s. 3)= limit 1 n=0 3) "asad an +1 3, Hence, by definition of the limit of a variable, it is seen that both: S,, and S;,4, are variables approaching 2 as a limit as the number of terms increases without limit. Summing up the first two, three, four, etc., terms of (4), the sums are found by (B) and (C) to be alternately less and greater than 3, illustrating the case when the variable, in this case the sum of the terms of (A), is alternately less and greater than its limit. In the examples shown the variable never reaches its limit. This is not by any means always the case, for from the definition of the témit of a variable it is clear that the essence of the definition is simply that the numerical value of the difference between the variable and its limit shall ultimately become and remain less than any positive number we may choose, however small. (4) As an example illustrating the fact that the variable may reach its limit, consider the following. Let,a series of regular polygons be inscribed in a circle, the number of sides increasing indefinitely. Choosing any one of these, construct the circumscribed polygon whose sides touch the circle at the vertices of the inscribed polygon. Let p, and B be the perimeters of the inscribed and circumscribed polygons of n sides, and C the circumference of the circle, and sup- pose the values of a variable x to be as follows: pe Cb Pee CO; hee el. Then, evidently, and the limit is reached by the variable, every third value of the variable being C. i 14. Division by zero excluded. 0 is indeterminate. For the quotient of two numbers is that number which multiplied by the divisor will give the dividend. But any number whatever multiplied by zero gives THEORY OF LIMITS 13 zero, and the quotient is indeterminate; that is, any number whatever may be considered as the quotient, a result which is of no value. a . oa: . ‘ & has no meaning, a being different from zero, for there exists no number such that if it be multiplied by zero, the product will equal a. Therefore division by zero is not an admissible operation. Care should be taken not to divide by zero inadvertently, ‘The following fallacy is an illustration. Assume that ‘Then evidently Subtracting 0%, Factoring, Dividing by a —b, But therefore or, ‘The result is absurd, and is caused by the fact that we di ed by a 0. 15. Infinitesimals. A variable » whose limit is zero is called an infinitesimal.” ‘This is written limit» = 0, or, v= 0, and means that the successive numerical values of v ultimately become and remain less than any positive number however small. Such a variable is said to become indefinitely small or to ultimately vanish. It limit v = 1, then limit (» — = that is, the difference between a variable and its limit is an infinitesimal. Conversely, if the difference between a variable and a constant is an infinitesimal, then the variable approaches the constant as a limit. 16. The concept of infinity (co). If a variable v ultimately becomes and remains greater than any assigned positive number however large, we say v increases without limit, and write limit v=+ 00, or, v=+o. If a variable v ultimately becomes and remains algebraically less than any assigned negative number, we say v decreases without limit, oud ate limit» =— 0, or, » ‘*Hence a constant, no matter how small it may be, is not an infinitesimal, 14 DIFFERENTIAL CALCULUS If a variable v ultimately becomes and remains in numerical value greater than any assigned positive number however large, we say v, in numerical value, increases without limit, or v becomes infinitely great,* and write limit v= 00, or, v= 00. Infinity (0) is not a number; it simply serves to characterize a particular mode of variation of a variable by virtue of which it increases or decreases without limit. 17. Limiting value of a function. Given a function f(z). If the independent variable z takes on any series of values such that limit «= and at the same time the dependent variable f(x) takes on a series of corresponding values such that limit f(z) =4, then as a single statement this is written lim 44) = and is read the limit of f(), as x approaches the limit a in any manner, is A, 18. Continuous and discontinuous functions. A function f(2) is said to be continuous for « =a if the limiting value of the function when approaches the limit a in any manner is the value assigned to the function for z=a. In symbols, if wat f@) =F@), then f(2) is continuous for x =a. The function is said to be discontinuous for x =a if this condition is not satisfied. For example, if Limit 72) = 0 the function is discontinuous for 2= a. ‘The attention of the student is now called to the following cases which occur frequently. *On account of the notation used and for the sake of uniformity, the expression v 4+ is sometimes read » approaches the limit plus infinity. Similarly, v += is read v approaches the limit minus infinity, and + is read v, in numerical value, approaches the limit infinity. ‘While the above notation is convenient to use in this connection, the student must not forget that infinity is not a limit in the sense in which we defined a limit on p. 11, for infinity is not a number at all. THEORY OF LIMITS 15 Case I. As an example illustrating a simple case of a function con- tinuous for a particular value of the variable, consider the function x @=5 For ¢=1, f(z) =f(1)=8. Moreover, if x approaches the limit 1 in any manner, the function f(e) approaches 8 as a limit. Hence the function is continuous for 2 =1. Case II. The definition of a continuous function assumes that the function is already defined for 2=a. If this is not the case, how- ever, it is sometimes possible to assign such a value to the function for z=a that the condition of continuity shall be satisfied. The following theorem covers these cases. Theorem. Lf f(x) ts not defined for x= a, and ' wr FC) =B, then f(2) will be continuous for x =a, if B is assumed as the value of f(@) for x=a, Thus the function v4 z—2 is not defined for 2 = 2 (since then there would be division by zero). But for every other value of 2, limit and pet D=4; limit 2°— 4 therefore ime Although the function is not defined for z= 2, if we arbitrarily assign it the value 4 for z= 2, it then becomes continuous for this value. A function f(2) is said to be continuous in an interval when it is continuous for all values of x in this interval.* ‘In this book we shall deal only with functions which are in general continuous, that is, continuous for all values of , with the possible exception of certain isolated values, our results in general being understood as valid only for such values of x for which the function in question is actually continuous. Unless special attention is called thereto, we shall as a rule pay no attention to the possibilities of such exceptional values of for which the function is discontinuous. ‘The definition of a continuous funetion f(z) is sometimes roughly (but imperfectly) summed up in the statement that @ sinall change in x shall produce « small change in f(z). We shall not consider funetions having an infinite number of oscillations in a limited region. r : 16 DIFFERENTIAL CALCULUS 19. Continuity and discontinuity of functions illustrated by their graphs. (1) Consider the function 2’, and let “4 yar. If we assume values for 2 and calculate the corresponding values of y, we can plot a series of points. Drawing a smooth line free-hand [ through these points. a good representation of the gen- cral behavior of the unction may be obtained. ‘This picture or image of the function is called its graph. It is evidently the locus of all points satisfying equation (A). Such a series or assemblage of points is also called acurve, Evidently we may assume values of x so near together as to bring the values of y (and therefore the points of the curve) as near together as we please. In other words, there are no breaks in the curve, and the function 2” is continuous for all values of 2. (2) The graph of the continuous function sinz is plotted by draw- ing the locus of 0 x . |O x y=sina. It is seen that no break in the curve occurs anywhere. (3) The continuous function e is of very frequent occurrence in the Calculus. If we plot its graph from y=e, (¢=2.718--) ¥ we get a smooth curve as shown. From this it is clearly seen that, 1 (a) when x= 0, Jmlt y=) =1; a (b) when 2>0, y(=c*) is positive and increases as we pass towards the right from the origin; (ce) when 2<0, y(=@) is still positive and decreases as we pass towards the left from the origin. 7 (4) The function log, is closely related to the last one discussed. In fact, if we plot its graph | x from oie it will be seen that its graph has the same rela- tion to OX and OY as the graph of e* has to OY and OX. THEORY OF LIMITS 17 Here we see the following facts pictured : (a) For #=1, log.« = log,1= 0. (b) For 2 >1, log, is positive and increases as « increases, (c) For 1>2>0, log, is negative and increases in numerical value as x diminishes, that is, !™* log 2 =— 0. (A) For x=, log.z is not defined ; hence the entire graph lies to the right of OY. hg (5) Consider the function 4, and set = If the graph of this function be plotted, it will be seen that as x approaches the value zero from the left (negatively), the points of the curve ultimately drop down an infinitely great distance, and as 2 approaches the value zero from the right, the curve extends upward infinitely far. The curve then does not form a continuous branch from one side to the other of the axis of ¥, showing graphically that the function is discontinuous for z = 0, but continuous for all other values of 2. (6) From the graph of _ 2 7 it is seen that the function Qa 1-2 is discontinuous for the two values 241, but continuous for all other values of 2. (7) The graph of yestane shows that the function tana is dis- continuous for infinitely many values of the independent variable z, namely, z=", where n denotes any odd positive or negative integer, (8) The function are tan & Bb DIFFERENTIAL CALCULUS has infinitely many values for a given value of 2, the graph of equation y= are tana consisting of infinitely many branches. If, however, we confine our- selves to any single branch, the function is continuous. For instance, if we say that y shall be the are of smallest numeri- cal value whose tangent is 2, that is, y shall take 2 limited to the branch passing through the origin, and the condition for continuity is satisfied. (9) Similarly, on only values between —7 and 7, then we are are tan, 2 is found to be a many-valued function. Confining ourselves to one branch of the graph of ye tan , we see that as 2 approaches zero from the left, y approaches the limit — Z, and as approaches zero from the right, y approaches the 2 PP ight, y apy limit + z Hence the function is discon- tinuous when 2=0. Its value for 2=0 can be assigned at pleasure. Functions exist which are discontinuous for every value of the independent vari- able within a certain range. In the ordinary applications of the Cal- culus, however, we deal with functions which are discontinuous (if at all) only for certain isolated values of the independent variable s such functions are therefore in general continuous, and are the only ones considered in this book. 20, Fundamental theorems on limits. In problems involving limits the use of one or more of the following theorems is usually implied. It is assumed that the limit of each variable exists and is finite. Theorem I. The limit of the algebraic sum of a finite number of vari- ables is equal to the like algebraic sum of the limits of the several variables. Theorem II. The limit of the product of a finite number of variables ts equal to the product of the limits of the several variables. Theorem III. The limit of the quotient of two variables is equal to the quotient of the limits of the separate variables, provided the limit of the denominator is not zero. THEORY OF LIMITS 1 Before proving these theorems it is necessary to establish the fol lowing properties of infinitesimals. 1) The sum of a finite number of infinitesimals ts an infinitesimal. ‘To prove this we must show that the numerical value of this sum can be made Jess than any small positive quantity (as e) that may be igned (§ 15). That this is possible is evident, for, the limit of each infinitesimal being zero, each one can be made numerically less than £ (nm being the number of infinitesimals), and therefore their sum can n be made numerically less than e (2) The product of a constant ¢ and an infinitesimal is an infinitesimal. For the numerical value of the product can always be made less than any small positive quantity (ase) by making the numerical value of the infinitesimal Jess than £. e (3) The product of any finite number of infinitesimals és an infinitesimal. For the numerical yalue of the product may be made less than any small positive quantity that can be assigned. If the given product contains » factors, then since each infinitesimal may be assumed less than the xth root of ¢, the product can be made less than e itself. (4 Ifo ts a variable which approaches a limit l different from zero, then the quotient of an infinitesimal by v is also an infinitesimal. For if limit » = J, and & is any number numerically less than J, then, by defini- tion of a limit, v will ultimately become and remain numerically greater than & Hence the quotient £, where ¢ is an infinitesimal, will ulti- 0 mately become and remain numerically less than 4, and is therefore by (2) an infinitesimal. Proof of Theorem I. Let vy ry Uy +++ be the variables, and Uy 2 ly «=> their respective limits. We may then write ly where ey € limit)? Adding ++ are infinitesimals (ie. variables having zero for a (A) OF yt BA DH GHGt ht =Car ett). 20 DIFFERENTIAL CALCULUS Since the right-hand member is an infinitesimal by (1), p. 19, we have, from the converse theorem on p. 18, limit 2, + 4,44: =4ththt--5 or, limit (v, + v, + ¥,+ -+-) = limit v, + limit v, + limit o,+---, which was to be proved. Proof of Theorem II. Let v, and v, be the variables, 2, and 2, their respective limits, and ¢, and ¢, infinitesimals; then wy=lte, . and Malt ey Multiplying, 2,= + &) at &) = Ut het bet ee or, (B) v,04— Uly= Le, + ye, + €6y+ Since the right-hand member is an infinitesimal by (1) and (2), p. 19, we have, as before, limit (0,»,) = U1, = limit », - limit », which was to be proved. Proof of Theorem IIE. Using the same notation as before, % 4+) (t8—2) » Lte L' Mite ly or, ©) % 4 lee % lL LG+s) Here again the right-hand member is an infinitesimal by (4), p. 19, if 1, #0; hence io 1, __ limit v, limit (-*)=7'=j55° », i, limit », which was to be proved. It is evident that if any of the variables be replaced by constants, our reasoning still holds, and the above theorems are true. 21. Special limiting values. The following examples are ofspecial importance in the study of the Calculus. In the following examples a>O and ¢#0 THEORY OF LIMITS 21 Written in the form of limits. Abbreviated form often used. wm limit & 23 i oS @® c-o= 0, 3 2, @ 2 = 0, e 4 eam 6) when a <1; a-*=+0, (6) when a <1; ate =0, ~ when a>1; a7=0. (8) when a>1; Geo. 9 log,2=+ 00, when a <1; log,0 =+ Ba Ba (lo) _ log,2=— oo, when a<1; log, (+ 0)=—a (11) Hm loge =— 00, when a>1; log,0 =— oo. (12) , Nt log, =+ 0, when a>1; — log,(+.0)=-+o0. ‘The expressions in the second column are not to be considered as expressing numerical equalities (co not being a number); they are merely symbolical equations implying the relations indicated in the first column, and should be so understood. limit sin x _ x=0 Let 0 be the center of a circle whose radius is unity. Let are AM = are AM' =a, and let UT and M'L be tangents drawn to the cirele at Mand M'. From Geometry, MPM! < MAM' < MIM’; a : 2 si 2 or Qsine ) a & 0 Taking the logarithm of both sides to the base e, we get log y =» log a, or, v aot dy Toga y? and from (C), § 48, relating to inverse functions, we get @ or, cs Ties (4) tog aa’ Since v is a function of « and it is required to differentiate a” with respect to x, we must use formula (A), § 42, for differentiating a function of a function, namely, dy _dy do dx dv dz Substituting the value of y from (A), we get a . =loga-a d dv i oS (a) = log a- av 2. FO = toga aS When a=e, ies and IX becomes dv 1X =(e)=e=. @ Jc Daer The derivative of a constant with a variable exponent is equal to the product of the natural logarithm of the constant, the constant with the variable exponent, and the derivative of the exponent. RULES FOR DIFFERENTIATING 49 46. Differentiation of the general exponential function. Let yous Taking the logarithm of both sides to the base e, log.y = v log.u, or, y=erey, Differentiating by formula Ixa, ym 4 log u) =w (22+ logu). x _ Zw = 00 B 4 tog uw. The derivative of a function with a variable exponent is equal to the sum of the two results obtained by first differentiating by VI, regarding the exponent as constant; and again differentiating by IX, regarding the function as constant. Let » =n, any constant; then X reduces to Cau a“ Qt) = nv ae But this is the form differentiated in § 40; therefore VI holds true for any value of n. Iususreanive Examene 1, Differentiate y = log (2? + a). a < (a? +a) dy _ a, Solution. ao Fra by VIIa =at+a) = 2 Ans, eta Inpusrrative Examrre 2. Differentiate y = log VI— aa. d tt ~at + Solution, 2 by VIIa ae at at a=) F(- 20) by VI a-ayt 2 Ans. * wcan here assume only positive values. 50 DIFFERENTIAL CALCULUS In.usrrative Exampve 8. Differentiate y = a=, Solution, YW —roga-02 4 gx) by IX = 6xloga-at, Ans. Intusrrarive Exampe 4, Differentiate y = be?+2*. ay _ Solution. lution, a a b— (ee? +2) Jet) by IV betes (et + at) by Xu = 2beet+s", Ans. Intusrative Examere 5, Differentiate y = x, Solution, u = eet é (0) + 24 loge: é e by X = ent + we loge e 7 exe( o loge). Ans. 47. Logarithmic differentiation. Instead of applying VIII and VIIa at once in differentiating logarithmic functions, we may sometimes simplify the work by first making use of one of the formulas 7-10 on p. 1. Thus above Illustrative Example 2 may be solved as follows: Ittusrrative Exampre 1, Differentiate y = log V1— 2. Solution, By using 10, p. 1, we may write this in a form free from radicals as follows : y= }log(1— 2), a fay ‘Then wie = by VIIa 1 —2¢ - + Ans 2 1-2 feat Inusrrative Exampre 2, Differentiate y = log Solution, Simplifying by means of 10 and 8, p. 1, y= 4 [log (1 + 2%) — log (1 — 2%). a a uy fae ay weet? gt?) by Vitra, ee de 2,142? T—a ’ x 22 = =e. ai i¢e@*i-e-i-aw “™ RULES FOR DIFFERENTIATING 51 In differentiating an exponential function, especially a variable with a variable exponent, the best plan is first to take the logarithm of the function and then differentiate. Thus Illustrative Example 5, p- 50, is solved more elegantly as follows: sar, Solution, ‘Taking the logarithm of both sides, Intusrrative Exampre 8. Differentiate logy = et loge. By 9, p.1 Now differentiate both sides with respect to z. wy S =e 2 (log) + loge t oy by VIII and V aot 4 loge-er, sed + loge. e, dy 1 Wo ge iy(L a « ees) =en(E+ Ing). Am Iniusrrative Exampre 4, Differentiate y = (42? — 7)2+-V#—6, Solution. ‘Taking the logarithm of both sides, logy = (2 +-V2" = 5)log (422 — 7). Differentiating both sides with respect to 2, cae a 5) 8% (422 — z= yatta 5) ge qt los de Ve Y _ (a2 — y+ vans [8A +VE = 8) , loeP= 1] Ans, a te ans In the case of a function consisting of a number of factors it i times convenient to take the logarithm before differentiating. Ituustrative Exampre 5. Differentiate Solution, Taking the logarithm of both sides, logy = 3 [log (@ — 1) + log (x — 2) — log (z — 8) — log (x — 4)]. Differentiating both sides with respect to 2, a 2-3 4] 227-102411 @-Ne@-HE-He—4" 227-102—11 . @-pte—a!@-ai@—4ai or, Ans. 52 DIFFERENTIAL CALCULUS EXAMPLES Differentiate the following : = loge + a). 2. y= log (ax +b). 4. y = log(a? +2). 5. y= log (3 — 22 + 5). 6. y = loga (2a +2). 7. y=aloge, 8. f(z) = loge’. 9. f(z) = log*z. Hinr. logte = (log2)®, Use first VI, »=loge, n=3; and then VIIa. 10. $@) = log +2. ra =—24 1. f(@) = loge + VIF a). F(t) = : vit¢e 12, © cox = aes, a7, Leitso = aaa, dz dx 13, © arte a4 etess, 1s, 4 ato = 1aree toga. dz ao 6 14, 2 ase = 8087 loga, 19, Lo =2slogh-b*. dx ds a 15. “tog (8 — 28) = 20. aos ¢ ) @ lt 16. “to = al. ay my we 22, ya Tt t2e, Y= 2log7- (a + 1) 7 +24, 23, y= et-2, 2eloge-c# ==, 24. y = log : ae ie +e 25. Stea-2] =e(1-22—-2%, d (#1 Qer a ae et . teeny = ner (ar + 2). °6 alan) @+m ne RULES FOR DIFFERENTIATING 53 Yo = @2"~1 (n+ zloga) y =a (loge +1). 1 2 (1~loga) | 32, yaar. : : z 33, y = alex, of =loga?. alosz—1, 34. f(y) =logy- ev. rwe=e (sy +3 : _ logs vg) _ 1— Slogs 35, f(0) = MES. Sy = ashe, oe 36. f(z) =log (log). IO) = say" vig) _ 40g? (log 2) ', F(z) = log* (lc le F’ _—_ 31. F(@) = log" (og 2). te) = See . (t) = log (logt a). (oo 38, 6 (2) = log (lost) +O = aes Hlogaftt¥, z _ 89. ¥(y) = log 7 vy) i-# po echteee Vet +2 Hint. First rationalize the denominator. 1 40. f(0) = Io I@= 41. y = ae, dy ya, WW _ (1 4 loga) 2%. 42. y= e*. a (1+ log 2) a: ct dy ‘e\= c 43. y=. &_ (2¥ (iogS —1). Sue a @) (0 z ) n\n ae 2) =(-)- —=n(- 14 log’ sv-() eon G(s 45. w= 0. == ve (Ene), dv v a\ ad _ [ay =(-). =—=(- —logt—1). 46. 2 () & () (log a — logt 1). 41. yam". $= ast n-1 (loge +1). : 1 48, y=0%. $= te (Ioge + lout +2)- dy syloga 54 DIFFERENTIAL CALCULUS 50, Differentiate the following functions : @) ge logz. © Z elogz, () Eos (+H (b) ce 1. () ge 8, Q Ee, (a? + 52). 0) zg oe @ gee vies. @) ce Fayette, o£ © Letra @+1? dy @ +I (Sat 14245) bly “Ermer ae @ +2) @ +3) Him. Take logarithm of both sides before differentiating in this and the following examples. 8. y= w= dy __ @=1)8 (722 + 802 — 97) (e—28@—9)F de 12(2—2)f@@—8)¥ 63. y=a2Vi—a(l+2). wu 24+a—528 (+ 2%) | dy Sa 55. y= 28 (a + 82)* (a— 22)%, ba, y= a-at 5a (a + 82)? (a— 22) (a? + 2ar—122%), z 48. Differentiation of sin v. Let y=sin v By: General Rule, p. 29, considering v as the independent variable, we have Fist Srer. y + Ay =sin(v +Av). Seconp Step. Ay =sin(v + Av) —sin o* Av\ Av =2 fons t = 2 (o+ 8) int If we take the, third and fourth steps without transforming the right-hand member, there results : A Ay _sin (v + Av)—sin » | Third step. cd 7. ‘Av ‘av Fourth step. te jr Which is indeterminate (see footnote, p. 46). tLet Anvtav A= vedo and Bao Bao Adding, IFB=Tt Subtracting, 4-B= Av 1 Av 1 Therefore 4(4+B)=04 82. dua- mya erefore F(A +B) = v4 pd Be S Substituting these values of 4, B, }(4+B),}(4—B) in terms ofv and Av in the formula, from Trigonometry (42, p. 2), sin A—sin B=2cos 4(4 4B) sin 4 (4-B), we get sin (»-+ Ar) sin v= 2 608 (v+ 42) sin S2- RULES FOR DIFFERENTIATING 55 A s sin Se Taro Srer. SY cos (+2) x=} 2 Fourtn Srep. dy =cos vw dy rimie (202 sine ml ( 2 Since v is a function of 2 and it is required to differentiate sin v with respect to a, we must use formula (A), § 42, for differentiating a function of a function, namely, mt by $20.2 and Bthee(oot)-or] dy _ yy we, ae du de Substituting value wy from Fourth Step, we get dy_ dv a =cosv a I oe Lin v= cos 0. The statement of the corresponding rules will now be left to the student. 49. Differentiation of cos v. Let y = cose By 29, p. 2, this may be written in (5-*) Differentiating by formula XI, 4 — on (§—») 4 (5-2) ao 2) alo emf) 2 a, =—sine®. te [sinc cos (Ze) asin, y 9.0.2] mm “ Zoos ») =—sin of, 56 DIFFERENTIAL CALCULUS 50. Differentiation of tan v. Let. y=tany. By 27, p. 2, this may be written sine "cosy Differentiating by formula VII, a cos 0 5- (sin ») — sin» 5 S (eos ») | da cos*y dvs dv costy 2 . os” or ry Fain v qe - cos*v dv dx a, dv =a ees d dv XU .. (tan v) = sec?v—- a z ax 51. Differentiation of cot v. Let y=cotv. By 26, p. 2, this may be written tan Differentiating by formula VII, i 4 (tan v) az tanty sec?y — =——— =~ osc S. tan?e oh d dv xm ws = (cot v) =—csctv—- V gloat n) = — eset 52. Differentiation of sec v. Let y=secr. By 26, p. 2, this may be written cosy RULES FOR DIFFERENTIATING 57 Differentiating by formula VI, ad = (cos 0 4 ee dx cos*v sino @ agents cos*e _ 1 sinv dv cos v cos v dx = secv tan o®. dx da dv a. = (secv) = secv tanv—- wv ae ee) 53. Differentiation of csc v. Let y=osen By 26, p. 2, this may be written a - Y= Sno Differentiating by formula Vit, _ 4 Ging a Gis dcx sin*v dv cos o dv =— escv cotv——- da d a w. = (escv) = — cscv cotv—-- a Frat) a 54, Differentiation of vers v. Let. y = vers v. By Trigonometry this may be written y=1—cose 58 DIFFERENTIAL CALCULUS Differentiating, an ae +. 4 (versy) = sinv® Xvi =. ag = ae In the derivation of our formulas so far it has been necessary to apply the General Rule, p. 29 (ie. the four steps), only for the following: a du, dv dw ' WL Qe o—wy= T+ 2-2. Algebraic sum, a dy, du Vv Fw) =uZ4 0H. Product ot _ 2 a [uw _ "ae “ae . vil Z()= FI cuotient a a de . vir Fog.) = loge =. Logarithm, a. a . xI EZ Gino) = cos 02. Sine. dy _ ay de, XXV Bowe Function of a funetion. XXVI ao Inverse functions. de te dy Not only do all the other formulas we have deduced depend on these, but all we shall deduce hereafter depend on them as well. Hence it follows that the derivation of the fundamental formulas for differentiation involves the calculation of only two limits of any difficulty, viz., limit sin» v=0 0 by § 22, p. 21 and Jimit 44 9)" By § 23, p. 22 RULES FOR DIFFERENTIATING EXAMPLES Differentiate the following : 1, y=sinaz®, ay oc Fe 7 C8 ae F(a) (v= az] = 2 az cos az. 2 y=tanvi—a. = sect Vina a_ at or [v= vi-z.] = see? VI—#- (1a) (1) sec? VI— 2vi-« 3. y= costs, ‘This may also be written y= cosa)’, dy a ve 2 3 (cos)? — (cosa) [v= cosa and n=3.] Scostz(—sinz) — 8sinz costa, 4. y=sinnesinnz, dy _ sin me (sina) + sinne (sin na) (w= sinnz and v= sint2] 59 by XI by XIII by VI by XII = sinne-n(singye-12 (sina) + sine cosne © (nt) by VI and XI =nsinne-sint-12 cost + nsinz cos nt nsint—1z (sin ne cose + cos nz sin) nsin"—tzsin(n + 1). 5. y =secan, Ans. {Y= a secas tan av. 6. y = tan (ax +b). 2 asec (ax +0). 1. 8 = cos8az. Ba sasindax. | 8. 8 = cot(2f +8). s —Atesc?(2t +8). 9. f(y) =sin2y cosy. 14. r= acos26. S'(y) = 2cos2y cosy — sin 2y siny. F(t) =— 10 cot 5x esc? 52. F/(6) = tan2g. 03g. in?t (8 cost — sin*t). 1 \ | I i 60 DIFFERENTIAL CALCULUS a a oa o 15, Zsinte = sin2e, 23. Z cont = Erin‘. d ; Cee 2 a. 16. £ costa? =— 6 costat sinat, aa, Fain 5 = — F008 5s 2 2 og at. 2 es0% = F ot 2 25, Lesinz = esinz cos. a 18, 2 aVeosds =— 28028, 26, 4 sin(logay = £80984), ds cos 28 dz © oe = a _ sec® log2) 19, FG a(l — e088) = 8. 27. 7, tan (log) ==—TES. a ao 68 . © (log cos) =— tan. . © asin? 2 = asin? 2 cos’ 20. 7, (log cos) =— tana, 28, asin? 5 = asin? cose aa. © (og tana) : 29, © sin (cos a) =— sin 008(c08 @) a dtr ~ sine * da . Co dtane-1_ |, 22. (og sin?z) = 2 cots. 30. nang = Sine + cosa as az ~ cose = ae a 9% y= Jogtan( +5) de cose 83. f(z) = sin (# + a) cos (x — a). I’ (@) = cos 2a. 34, y= alana, natenns sec? na log a. y = @m= (cosa — sin?2). (cot + log sinz). 35. y = ese sing. 36. = et log sing, 87. Differentiate the following functions: (@) Z sin 5a? @ é ese (log). e) g ea beort, (b) Ecos a — ta). ) awsae, @ g jeoste (O) cian. (n) Loos! (loga). (m) Foor d. @ Z cot Vaz. i) Stant vi (a) BVI cor. @ Z seced=, Oe log (sin? az). (0) E tog VI= Binks 38. é (anesinay ar-1etinz (n 4 2 082). 39. ce cosmz) = e# (1 cos me — msin mz). _ 14 0088 oo aan Oe coad TO=— Green di 41. Ig) = Plese= 89), F'($) = ent sing. 42. f(s) = (scots)?. F'(8) = 2s cots (cots — s cscs). RULES FOR DIFFERENTIATING 61 43, r=} tan®O— tand +8. g- ant 6. = ong, wy ine (Sat ). 44, y = ang, = ans (SRE + loge cos 45, y = (sin2)= y' = (inz)= [log sinz + zcotz). 46. y = (sina)nz, * y= (sin z)"02(1 + sec? log sine) a 47. Prove — rove 5 cos =- sino®, using the General Rule. 48, Prove Zcotv =- eset by replacing cot » by = 55. Differentiation of arc sin v. Let, y =are sin v;* then v=siny. Differentiating with respect to y by M1, oe ay 83 dy_ 1 h yi it , pe Al therefore ueoney By (C), p. 46 But since v is a function of 2, this may be substituted in (A), p. 45 giving 1 ad Vi-vde [x yaVizsinty =VI= a, the positive sign of the radical betng | singe cosy is positive for all values of y between — 3 and inclusive, a ‘ XVI a. (are sin v) vi _ ) vi-v * Tt should be remembered that this fnnetion is defined only for values of » between —1 and +1 inclusive and that y (the function) is many-valued, there being infinitely many ares whose sines all equal v. ‘Thus, in the figure (the locus of y=are sin v), when ¥= OM, y= MP1, MPa, MPs,-~, MQuy, MQoy-++- In the above discussion, in order to make the funetion single-valued, only values of y between — and 7 inclusive (points on are QOP) are considered ; a i ees 62 DIFFERENTIAL CALCULUS 56. Differentiation of arc cos v. Let y =are cos 0; * then v= cosy. Differentiating with respect to y by XI, dv . erty therefore # ay By (€), p. 46 But since v is a function of 2, this may be substituted in the formula (A), p. 45 giving XIX = (are cos v) = dx 1-¥ 57. Differentiation of arc tan v, Let y=are tan v;t then v=tany. Differentiating with respect to y by XIV, ® sect ys dy ys dy a therefore S¥ ag By (€), p. 46 * This function is defined only for values of » between ~1 and +1 inclu- sive, and is many-valued. In the figure (the locus of y= arc cos), when v=OM, y= MP3, MPs,+**, MQq, M Qa, In order to make the function single-valued, only values of y between © and ® inclusive are considered; that is, the smallest positive are whose cosine is 2. Hence we confine ourselves to are QP of the graph. 4 This function is defined for all values of », and is many- valued, as is clearly shown by its graph. Tn order to make it, single-valued, only values of y between ~ 5 sidered; that is, the are of smallest numerical value whose tangent is v (branch 408). © and 5 are con- RULES FOR DIFFERENTIATING 63 But since v is a function of 2, this may be substituted in the formula di dy dv Z 2.2, (A), p. 45 giving yt dz secy dz 1d 1+ dr [seoty =14 tanty=14 v4] dv d dx Sa tan v) =——- wx gear tan) = 58. Differentiation of arc cot v.* Following the method of the last section, we get dv : d a XX (are cot v) = ———. a » 14+v 59. Differentiation of arc sec v. Let y =are sec v3t then vasecy. * ‘This funetion is defined for all values of », and is many-valued,-as is seen from its graph (Fig. a). In order to make it single-valued, only values of y between 0 and m are considered; that is, the smallest positive are whose cotangent is v. Hence we confine our selves to branch 4B. Fie.a Fie.d } This function is defined for all values of » except those lying between ~1 and +1, and is seen to be many-valued. ‘To make the function single-valued, y is taken as the are of smallest. numerical value whose secant is v. ‘This means that if v is positive, we confine ourselves to points on are AB (Fig. b), y taking on values between 0 and 5 (Omay be included) ; and if v is negative, we confine ourselves to points on are DC, y taking on values between ~ and - 5 (Cm may be included). 64 DIFFERENTIAL CALCULUS Differentiating with respect to y by XV, Fiasco y tan ys dy 1 therefore By (C), p. 46 dy secytany But since v is a function of 2, this may be substituted in the formula dy_dy d SY YS, ‘A), p. 45 de dv de oe ivi, ao giving de secy tany dx =a oVe—l de eye ad any = Seecty sed ha plas ian or a radieal being taken, since tan y is positive for all values of y dotweon Oand and between — wand —F including and —7. 2 beets oe (are sec v) = bad "de vv a1 60. Differentiation of arc csc v.* Let y =are csc v3 then v=cscy. Differentiating with respect to y by XVI and following the method of the last section, we get gle d AMIE = — (are csc v) = - ———- a 2 vv 1 = __* This function is defined for all values of v except those lying between — 1 and +1, and is seen to be many-valued. To make the function single-valued, y is taken as the are of small- ‘est numerical value whose cosecant is v. ‘This means that if » is positive, we confine ourselves to points on the are AB (Fig. a), y taking on values between 0 and = G may be included); and if v is negative, we confine ourselves to points on the are CD, y taking on values between ~ 7 and — 3 (- 7 may be included) RULES FOR DIFFERENTIATING 65 61. Differentiation of arc vers v. Let y = are vers v; * then v=vers y. Differentiating with respect to y by XVII, therefore By (€), p. 46 But since v is a function of 2, this may be substituted in the formula ay _ aya, ee dz dv de (A), p- 45 vin, oe | de siny de d 5 ido, V2y— de [en ee the plus sign of the ral] being taken, since sin y is positive for all values of y between 0 and © inclusive, dv xxv 2 (are vers v) = ade - vav—-# EXAMPLES Differentiate the following : 1. y= arc tanaz*, Solution. by XX ae T+ Cae (v= art] 2ax “T+ at 2. y=aresin(@x— 428), Solution. # by XVIII [waz 420) - 8-122? 8 “Vi-te+aat—ioe Ving * Defined only for values of » between 0 and 2 inclusive, and is many- valued. To make the funetion continuous, y is taken as the smallest positive are whose versed sine is v; that is, y lies between 0 and x inclusive. Hence ‘we confine ourselves to are OP of the graph (Fig. a). 66 DIFFERENTIAL CALCULUS wt. 3 y=aresee— 2 Solution. a oe [eH ei (F l — a Vai a —22 5. — are cot (2? — 5) = ————_.. a Ce aie; po cian ee ee dz 1-2 142 a 1. © are cosee a a 2 8. —are vers 22? = o a ea 14. f(a) = 2 Vo? — @ + a? are sin 15. f(e) = Ve —# + aaresin®. 16, z= rare vers! — V2ry— 17. 6 = are sin(r—1). 18. ¢= are tan7 +4, 1—ar 1 19. s = are see 1—# d ina) = (ware sin 2) = are sing + a. Hltano are tan 8) = sec? are tan @ + “8% 22. $ Dog (are cos t)] = 23. f(y) = are cos (logy). 24. f(8) = aresin by XXII a i meres 12, “are tan ae 13, Laresin2 2 v2 I(t) =2Ve—2. se =(452) a+, e dy ao ar ay ar as tand 146 S'()= AVI + c8cd. RULES FOR DIFFERENTIATING 67 25. f(¢) = are tan. eo S’()= }. rotan 26. p = ewetsna, ale dg +? 27. w= aretan® 2". a do eee 28, 8 = are cos oe dat +e oo yarns (bane nes) e } Vow 30, y = “are tana. va or[ ag terete tana (lt tees) | 31. y = aresin (Sinz). y=1. 4sinz 4 32, y = are tan ————_. (= . : B+ Boose Y= 548 cose . 33, y = are cot! + log, |® os © nea a—at 34, y = log (43, J arctan. a 35. y = VI—@ are sine — yan 36. Differentiate the following functions : 4 are sin 22? 2p aresin’ (a) Faresin 2a, (f) Fe aresing. da da a 2 garctanct, (b) Faretan ate, ae . © Zare seoe. (ny om g?-are tan gt. @ Zaare cosa. @ Garesind’. (@) 2 at are cotaz. @) SaretanvIe ® az” * aw . zaresing ® a sinVI-#. () 4 are tan (log a2). ae a 8 (0m) gy (at + S)are see. Qa 8 (ny = arc cot. (0) Z VI=@aresint. Formulas (A), p. 45, for differentiating a function of a function, and (C), p. 46, for differentiating inverse functions, have been added to the list of formulas at the beginning of this chapter,as XXV and XXVI respectively. ay In the next eight examples, first find u and substitute the results in ° * * to fina &. de by differentiation and then by XXV + As was pointed out on p. 44, it might be possible to eliminate v between the two given expressions so as to find y directly as a function of z, but in most cases the above method is to be preferred. 68 DIFFERENTIAL CALCULUS In general our results should be expressed explicitly in terms of the independent ! variable; that is, a in terms of 2, $ in terms of y, 2 in terms of 8, etc. 81, y=2—4,0= 82241. WW < 4p; @ — 6a, substituting in XXV, av a 7 38. y= tan2v, v= aretan(2¢—1), ; substituting in XXV, +1 dy Qsoct2v _» tan?2Qv4+1 _ 2et— 2241 @ 2-241 22-2241 2@—ay? et | [sinco v= ato tan 22—b, tan y= 22-1, an 20m 22=L, | 39. y= 309-4045, v= 22*— 5, Me raa8 — sot, | iz 20 dy 40. y= 10s ove Y= Fy—8 Qa— ax @— | A dy 41. y = log (a? — v2), v= asing. qen7 Btn. I dy e | 42, y =arctan(a+v), v= 6 jad ae | : | a 1+ are? 43, r= e884 et, 8 = log(t— 2). fo4e—oe st. In the following examples first find $ by differentiation and then substitute in wot by XXVI to fina 2, dy 44,c=yVIty. a 2vity_ 45, 2= Vivo. oy. 2Vi toy a siny 46. 5= Teelosv oa oe 41. 2 = alogt t¥e—¥ venr, ay __ Vea ¥ 48. ¢ = rare vers! — Vary =, 49, Show that the geometrical significance of XXVI is that the tangent makes complementary angles with the two codrdinate axes. RULES FOR DIFFERENTIATING 69 62. Implicit functions. When a relation between z and y is given by means of an equation not solved for y, then y is called an implicit function of x For example, the equation 2—4dy=0 defines y as an implicit function of 2. Evidently z is also defined by means of this equation as an implicit function of y. Similarly, e+y+2—ae=0 defines any one of the three variables as an implicit function of the “other two. It is sometimes possible to solve the equation defining an implicit function for one of the variables and thus change it into an explicit function. For instance, the above two implicit functions may be solved for y, giving 2 Jag and y=tVe—2—2; the first showing y as an explicit function of 2, and the second as an explicit function of z and z. In agiven case, however, such a solution may be either impossible or too complicated for convenient use. ‘The two implicit functions used in this article for illustration may be respectively denoted by ¢(n, y= 0 and Fay )=0. 63. Differentiation of implicit functions. When y is defined as an implicit function of x by means of an equation in the form 1c) S@N=% it was explained in the last section how it might be inconvenient to solve for y in terms of x; that is, to find yas an explicit function of « so that the formulas we have deduced in this chapter may be applied directly. Such, for instance, would be the case for the equation (B) aa’ + 22'y —y'x—10 = 0. We then follow the rule: Differentiate, regarding y as a function of x, and put the result equal to zero.* That is, © Ere n=0. process will be justified in §7. Only corresponding values of # and y which given equation inay be substituted in the derivative. 70 DIFFERENTIAL CALCULUS d, Let us apply this rule in finding on from (B). dx SF (att 22ty— yfr—10)=0; by (€) J @ty+ Lary-Lyn—-Lany=o; 5 dy dy ax? + 229% —y Tay +200 + 6aty— yf Tay 0; Ba =T ayy Uo yf — 6 a2 — 6 2°ys dy _y'—6ax'—~6a°y Beer The student should observe that in general the result will contain both « and y. EXAMPLES Differentiate the following by the above rule : Ly =4pe. 2, » 2 x 10. 12, 13, 14, aeyar. Dat + aty? = att, By +2ar=0, ata yt oat. Baytaat. n> van +6 . yt — Day += 0. . a8 + > — Bary = 0. may, » p? = a? cos 20, pt cosd = a? sin86, cos (uv) = ev. 8 = cos (0+ 4). de Pa at dy _y—aylogy dz x? — zy loge dp __atsin28 ae : dp _ 8a*cos36 + psing do 2p cosd . du _ c+ usin (w) dp ~ = vsin(uv) ao sin (0+ ¢) de ~T+sin@+ RULES FOR DIFFERENTIATING 71 15, Find 2 from the following equations: (a) 2? = ay. (f) yt+yt+4c=0 (k) tanz + y> (b) a? 4 4y? = 16, (g) yet —y = 5. () cosy + 8a? (0) 2? — aty? = a, (h) 2? — 24 = 98, (m) zeoty+y @ x=a+a. (i) ay + 4y= (n) y? = loge. () ®@-y=16. (i) Y =sin2a, (0) +278 =0, 16. A race track has the form of the circle 2? + y# = 2600, The directions OX and OY are east and north respectively, and the unit is 1 rod. If a runner starts east at the extreme north point, in what direction will he be going (a) when 25-V2 rods east of OY? Ans, Southeast or southwest, (b) when 25 -V2 rods north of OX? Southeast or northeast. (c) when 30 rods west of OF? E, 86° 52/ 12” N. or W. 8652/12” N. (a) when 40 rods south of OX? (e) when 10 rods east of OF ? 17. An automobile course is elliptic in form, the major axis being 6 miles long and running east and west, while the minor axis is 2 miles long. If a car starts north at the extreme east point of the course, in what direction will the car be going (a) when 2 miles west of the starting point? (b) when 3 mile north of the starting point? MISCELLANEOUS EXAMPLES Differentiate the following functions : 1, aresin-VI Ans. 42° 2. xer, oF (22% +1). 3, log sin = Scot 8 22 4, are cos”. a y roe a @-ai 6 loge * T+ log (1+ log 2)?" 7. log see (1 — 22). —2tan(1— 22). 8, wet 3z, ne-82(2— 82), 9. tos.4/ cost, eset. 1+ cost 10, arc sin -V¥ (1 — cos). . a 28 2 11. are tan — —_————. ‘s (= 58) Vs? 1 12. @2—-1),[ 2. 44a of 2. lta 38042) Vite yg, Maresing , (e+ 2)V1— at, @aresina. 72 u 16. WW. 18. 19. 20. 21. 2% 25. 26. 27. 28. 29. 30. s s DIFFERENTIAL CALCULUS sed + tet. are tan } (@* 4-32), a2 ( ine, @+ahee—nt al eeett—82), are tan V1— a, 2 cosz gana, log sin? 44. = log sin ax. sin? 9 cos g. a 2V0= com mia onectane tm Vi+¢at tan? a — log sec?sr, Blog (2 cose + 3sine) + 2a 13 a are cot + log 31. 32. 33. 84, 35. 36. 37. 38. 39. 40, 41. 42, 43. 44, 45. 46. 47. «+ (log tan 3 — 22), anaes t (+2029 242) @+2)@—8z) . are tan (log 82). . VO= anny, log V (a — bam, log fA +1. y-l enresse20, (esa 1+4z cose e log sinz, are sin arc tan a. atime, cot (log az). 1 (1— 82%) e*, vi-w log. ‘ Vite CHAPTER VI SIMPLE APPLICATIONS OF THE DERIVATIVE 64. Direction of a curve. It was shown in § 82, p. 31, that if y=f@) is the equation of a curve (see figure), then #.. tan = slope of line tangent to the curve at any point P. The direction of a curve at any point is defined to be the same as the direction of the line tangent to the curve at that point. From this it follows at once that ay an tant = slope of the curve at any point P. At a particular point whose coérdinates are known we write (z = slope of the curve (or tangent) at point (x, y,). c= v= At points such as .D, F, H, where the curve (or tangent) is parallel to the avis of X, oy 7 =0°; therefore Fae At points such as A, B, G, where the curve (or tangent) is per- pendicular to the axis of X, dy 7 = 90°; therefore 2 =o. dx 8 74. DIFFERENTIAL CALCULUS. At points such as Z, where the curve is rising,* 7 =an acute angle; therefore # =a positive number. The curve (or tangent) has a positive slope to the left of B, between D and F, and to the right of @ At points such as C, where the curve és falling,* a + =an obtuse angle; therefore S =a negative number. ‘The curve (or tangent) has a negative slope between B and D, and between F and G. 2 Tnuusrrative Exampre 1, Given the curve y = . —a42 (see figure). (a) Find + when # =1. (b) Find 7 when 2 = 3, (©) Find the points where the curve is parallel to OX. (a) Find the points where + = 45°, (e) Find the points where the curve is parallel to the line 20— By =6 (line AB), Solution. Differentiating, au = 2 — 2 = slope at any point, (a) tan -|% —2=-1; therefore r= 135°, Ans, (b) tanr -[2 —6=8; therefore 7 = are tan3, Ans. () T=0% tant= & =0; therefore 2? 22= 0, Solving this equation, we find that 0 or 2, giving points C and D where the curve (or tangent) is parallel to OX. ty Solving, we get « =14-V2, a) 45%, tant 1; therefore 2? — 22 giving two points where the slope of the eurye (or tangent) is unity. (e) Slope of line = 3; therefore 2* — 2% = 3. Solving, we get 2 =1-4. points Z and F where curve (or tangent) is parallel to line AB. giving Since a curve at any point has the same direction as its tangent at that point, the angle between two curves at a common point will be the angle between their tangents at that point. Inwusrrariy. Vind te angle of intersection of the circles (A) e+y—4e51, (B) e+y—2y * When moving from left to right on curve. 75 SIMPLE APPLICATIONS OF THE DERIVATIV Solution. Solving simultaneously, we find the points of intersection to be (8, 2) and te fy 2-2 rom (4). By § 63, p. 69 yy —*_ from (B). By § 68, p. 69 — — } = slope of tangent to (A) at (3, 2). — 8 = slope of tangent to (B) at (8, 2). ‘The formula for finding the angle between two lines whose slopes are m, and m, is aa 55, p. 3 Substituting, tan @ = oe =1; therefore 9 = 45°. Ans. ‘This is also the angle of intersection at the point (1, — 2). tang = EXAMPLES ‘The corresponding figure should be drawn in each of the following example: 1. Find the slope of y = the origin, Ans. 1= tant. i+a 2. What angle does the tangent to the curve 2%y? = a¥(e + y) at the origin make with the axis of X? Ans. 7 = 135°, 3. What is the direction in which the point generating the graph of y= 38a?—x tends to move at the instant when z=12? Ans. Parallel toa line whose slope is 5. 4, Show that & (or slope) is constant for a straight line. in 5. Find the points where the eurve y= 2° — 32° — 9x + 5 is parallel to the axis of x. Ans, 2 =8,2=—1. 6. At what point on y? = 22° is the slope equal to 3? Ans. (2, 4). 7. At what points on the circle a? + y? = 1? is the slope of the tangent line equal a . ane, (45, 22). 8. Where will a point moving on the parabola y = 2*— 7. +8 be moving paral- lel to the line y = 5242? Ans. (6, — 8). 9. Find the points where a particle moving on the civele a® + y2 = 169 moves per- pendicular to the line 52 + 12y = 60. Ans, (+12, 5). 10. Show that all the curves of the system y = log kr have the sume slope ; i.e. the slope is independent of k, 11. The path of the projectile from a mortar cannon lies on the parabola y= 2x —2a?; the unit is 1 mile, OX being horizontal and OY vertical, and the origin being the point of projection, Find the direction of motion of the projectile (a) at instant of projection ; (b) when it strikes a vertical cliff 1) miles distant. (c) Where will the path make an inclination of 45° with the horizontal ? (a) Where will the projectile travel horizontally ? Ans. (a) are tan 2; (b) 185°; (¢) Gi. #5 (@) GD. 6 DIFFERENTIAL CALCULUS 12, If the cannon in the preceding example was situated on a hillside of inclination 45°, at what angle would a shot fired up strike the hillside ? Ans, 45°. 18. At what angles does a road following the line 8y — 2 — 8 = O intersect a rail- way track following the parabola y? = 82. ‘Ans. arctan}, and arc tan}. 14. Find the angle of intersection between the parabola y? = 6x and the circle ayy =i6. Ans, are tan§ V8. 2 yt 15. Show that the hyperbola 2° —y? = 5 and the ellipse + e =1 intersect at right angles. = 16. Show that the circle 2? 4 y? = 8 ax and the cissoid y* = (a) are perpendicular at the origin ; (b) intersect at an angle of 45° at two other points. 2B 2a—z 17. Find the angle of intersection of the parabola 2?=4ay and the witch 8a* Bae Ans, arctan 8 = 71° 88.9, 18. Show that the tangents to the folium of Descartes 2° + y° = 3 azy at the points where it meets the parabola y* = az are parallel to the axis of Y. 19. At how many points will a particle moving on the curve y = «8 — 22% 42—4 be moving parallel to the axis of X? What are the points ? Ans. Two at (1, — 4) and (, — 484). 20. Find the angle at which the parabolas y = 84? — 1 and y = 2? 42 intersect. Ans. are tan fy. 21, Find the relation between the coefficients of the conics a,2* + by? =1 and az? + byy? = 1 when they intersect at right angles. iota a a by 65. Equations of tangent and normal, lengths of subtangent and subnormal. Rectangular codrdinates. The equation of a straight line passing through the point (z, y,) and having the slope, m is y-y=m@e—2,). 54, (0), p.8 If this line is tangent to the curve AB at the point P(x, y,), then from § 64, p. 73, m=tan Tt Hence at point of contact R(a, y,) the equation of the tangent line TR is i wy, () y-W=Te- 4H): dx, * By this notation is meant that we should first find a, then in the result substitute a for x and y; for y. ‘The student is warned against interpreting the symbol mm to mean the 1 derivative of 1 with respect to 7, for that has no meaning whatever, since #, and y; are doth constants. SIMPLE APPLICATIONS OF THE DERIVATIVE TT The normal being perpendicular to tangent, its slope is 1_ dy, ae By 55, p.3 And since it also passes through the point of contact B(x, y,), we have for the equation of the normal RN dr, @) y-Ww=—- (4-H). dy, That portion of the tangent which is intercepted between the point of contact and OX is called the length of the tangent (= TR), and its projection on the axis of X is called the length of the subtangent (=7M). Similarly, we have the length of the normal (= BN) and the length of the subnormal (= MN). i" a In the triangle 7RM, tan r= 2 TM ; therefore _ tant @) rM* y, 21 = tength of subtangent. dy, In the triangle MBN, tan r= ay, therefore LR? (4) MNt =MP tan t=y, # = length of subnormal. 4 The length of tangent (= 7P) and the length of normal (= RN) may then be found directly from the figure, each being the hypotenuse of a right triangle having the two legs known. Thus een TP, =V TM + UR = NCE) +, : dA? 6) =y, (3) +1=length of tangent. i 1s dz, ay. 6 = Ht +(F) = length of normal. fy The student is advised to get the lengths of the tangent and of the normal directly from the figure rather than by using (5) and (6). When the length of subtangent or subnormal at a point on a curve is determined, the tangent and normal may be easily constructed. * Uf subtangent extends to the right of T, we consider it positive; if to the left, negative. + If subnormal extends to the right of a, we consider it positive; if to the left, negative. 78 DIFFERENTIAL CALCULUS. EXAMPLES 1, Find the equations of tangent and normal, lengths of subtangent, subnormal 3 tangent, an/l normal at the point (a, ) on the eissoid 9? = = = ane 3 Solution. cae : es de ~y@a—ap dy, [dy Baa . Hence w= [z sop aaa a 2 = HORE of tangent. ¥ ies Substituting in (1) gives y= 2a — a, equation of tangent. ie Substituting in (2) gives x 2y +2 =a, equation of normal, opr Substituting in (8) gives T= 5 = length of subtangent. Substituting in (4) gives MN =2a _ Also PT = V(THY + MPF = Vite +a length of subnormal. ae = length of tangent, and PN =V(MN} + (MPP = Vie + 2 5 = length of normal. 2. Find equations of tangent and normal to the ellipse 2? + 2y* — 22y — 2 =O at the points where x = 1. Ans, At (1,0), 2y=e—l,y+2e=2 At(1,1),2y=e41y42e=3. 8. Find equations of tangent and normal, lengths of subtangent. and subnormal at the point (2,, y,) on the circle a? + y? = 12. Ans. 22+ AY =", ty — yt =0, — 4, Show that the subtangent to the parabola y? = 4 pz is bisected at the vertex, and that-the subnormal is constant and equal to 2p. 5. Find the equation of the tangent at (2,, 7,) to the ellipse = @ 3 6. Find equations of tangent and normal to the witeh y= we the point wherez = 2a, ae Ans, 24 2y=4a,y 22-84. 7. Prove that at = point on, the eatenary y ge 4e74 the lengths of sub- normal and normal are? (ea. Sie) ae a 8, Find equations of tangent and normal, lengths of subtangent and swbnormal, to cach of the following curves at the points indicated : (a) y= a2 at (1, 4). (©) y= 9—22at (— 3,0). () y= 4eat (9,— 6). (f) @ = Gy where z (c) 2 + Sy? = 14 where y = 1. (g) a —ay + 22—-9=0, (8,2). (@) 224 y= Hat (—3,—4). (h) 22? y= 14 at B, — 2). “In Exs. 8 and 5 the student should notice that if we drop the subscripts in equations of tangents, they reduce to the equations of the curves themselves. SIMPLE APPLICATIONS OF THE DERIVATIVE 79 9, Prove that the length of subtangent to y = a* is constant and equal to ; og a 10. Get the equation of tangent to the parabola y? = 20x which makes an angle of 45° with the axis of X. Ans, y=ut5. Hin. First find point of contact by method of Illustrative Example 1, (4), p. 74. 11, Find equations of tangents to the cirele 2? + y? = 62 which are parallel to the line 22 + 8y =6. Ans, 22+ 3y + 26=0. 12. Find equations of tangents to the hyperbola 42?— 9y? + 86 = 0 which are perpendicular to the line 2y + 5a = 10. Ans, 20 —5y4+8=0, 18. Show that in the equilateral hyperbola 2y =a? the area of the triangle formed by a tangent and the codrdinate axes is constant and equal to a’. 14, Find equations of tangents and normals to the curve y?=2a?— a at the points where z = 1. Ans. At(1,1),2y=241,y +22 =3. At(1,—1),2y —ly-22=— 15. Show that the sum of the intercepts of the tangent to the parabola dt ytoat on the codrdinate axes is constant and equal toa. 16. Find the equation of tangent to the curve 2? (x + y) = a? (x — y) at the origin. Ans. y =a. 17. Show that for the hypocycloid 28 + y$ = al that portion of the tangent included between the cobrdinate axes is constant and equal to a, 18. Show that the curve y = ae* has a constant subtangent. 66. Parametric equations of a curve. Let the equation of a curve be (A) Fay)=0. If « is given as a function of a third variable, ¢ say, called a param- eter, then by virtue of (A) y is also a function of ¢, and the same fune- tional relation (4) between x and y may generally be expressed by means of equations in the form : : z=fO» oy y= 9s each value of ¢ giving a value of z and a value of y. Equations (B) are called parametric equations of the curve. If we eliminate ¢ between equations (B), it is evident that the relation (4) must result. For example, take equation of circle Pyar, ory=Vr Let a=rcost; then y=rsint, and we have r=rcost, Cc fi {yr rSine as parametric equations of the circle in the figure, ¢ being the parameter. VS 80 DIFFERENTIAL CALCULUS If we eliminate ¢ between equations (C) by squaring and add- ing the results, we have 2+ y= 7 (cos*t + sin®t)= 7", the rectangular equation of the circle. It is evident that if ¢ varies from 0 to 27, the point P (2, y) will describe a complete circumference. In $71 we shall discuss the motion of a point P, which motion is defined by equations such as z=fO, y=oO- We call these the parametric equations of the path, the time ¢ being the parameter. Thus in Ex. 2, p. 93, we see that T= 0, COSa-t, y=—hot +, sina-t are really the parametric equations of the trajectory of a projectile, the time ¢ being the parameter. ‘The elimination of ¢ gives the rectan- gular equation of the trajectory ge =a tana ——*_.. y Due cosa Since from (2B) y is given as a function of ¢, and ¢ as a function of a, we have 2 dy dy dt aa ae ae by Xx by XXVE that is, dy ® a Hence, if the parametric equations of a curve are given, we can find equations of tangent and normal, lengths of subtangent and subnor- mal at a given point on the curve, by first finding the value of 2 at that point from (D) and then substituting in formulas (1), (2), (3), (4) of the last section, SIMPLE APPLICATIONS OF THE DERIVATIVE 81 Intusrrative Examere 1. Find equations of tangent and normal, lengths of sabtangent and subnormal to the ellipse P (x =acos¢, ©) Ly =bsing* at the point where ¢ = Z 7 j ae : Solution, The parameter b apt tn, z = beos¢. : dy __ deose Substituting in (D), . = slope at any point. ubstituting in (D), SH =— ZEB — slope at any poin Substituting ¢ == in the given equations (E), we eet (Fe Za) as the point of v2 V2, ss contact. Hence Substituting in (1), p. 76, ” or, br + ay =V2ab, equation of tangent, > Substituting in (2), p. 77, y— 3(e- @ ) = v2} v2, or, ‘V2 (az — by) = a? — b°, equation of normal. Substituting in (8) and (4), p. 77, d ( ») a — 2) =— = tength of subnormal, : v2\ a av2 ab (- *) — = length of subtangent. ~ va) 5 ; *As in the figure draw the major and minor auxiliary circles of the ellipse. Through two points B and C on the same radius draw lines parallel to the axes of codrdinates. ‘These lines will intersect in a point P (x, y) on the y| ellipse, because 2=0A= OB cosp~acosd and y= AP = OD= OCsing=bsing, on, cos and = sing. a Now squaring and adding, we get y cos? + sin? 6=1, the rectangular equation of the ellipse. ¢ is sometimes called the eccentric angle’of the ellipse at the point P. 82 DIFFERENTIAL CALCULUS Iuwsrrarive Exampie 2. Given equation of the eycloid * in parametric form z=a(—sind), t = a(1— cos), 4 being the variable parameter; find lengths of subtangent, subnormal, tangent, and normal at the point where =f. Solution. ae a(1—cos6), uy = asin. Substituting in (D), p. 80, dy _ _8iN6__ stope at any point. dz 1—cos@ ~ Since @ =, the point of contact is & -4, 4), and ze Substituting in (8), (4), (6), (6) of the last section, we get length of subtangent length of subnormal = length of tangent, v2, length of normal EXAMPLES Find equations of tangent and normal, lengths of subtangent and subnormal to each of the following curves at the point indicated : Tangent Normat Subt. Subn. ‘Lea t2y L e—dy41=0, 8242y-9=0, 2% 4 Ba=,yaBs t= 122—y—16=0, t4+12y—98=0, 3, 96. 3 2=By=8; 82—2y—-1=0, 2248y—-5=0, % 4. g=2e,y ert; t z+2y—4=0, 22—y—-3=0, -h 5. @=sint, y= cos2t; t=. 2y44e—8=0, 4y—20— * The path described by a point on the circumference of a cirele which rolls without sliding on a fixed straight line is called the eycloid. Let the radius of the rolling cirele be a, P the generating point, and Af the point of contact with the fixed line OX, which is called the base. If are PM equals O3fin length, then P will touch at 0 if the circle is rolled to the left. ‘We have, denoting angle POM by 6, M-NM= a6-a sin 0=«(9-sin 6), PN= MC- AC=a-aeos = a(1~c0s 6), the parametric equations of the cycloid, the angle @ through which the rolling circle turns Deing the parameter. OD=27a is called the base of one arch of the eycloid, and the point V is called the vertex. Eliminating 6, we get the rectangular equation eaasieea(=2) VF. SIMPLE APPLICATIONS OF THE DERIVATIVE 83 Ly=e; lL. «= cost, y= sin2t; 1. e=8t,y=6t—-#; t=0. 12. 2 =B8e-4 y= Bet; tH 0. =@jy=t;t=2 13, 2 =sint, y =2cost; t 9 e=0,y=2;t=—-1. 14, = dost, y = Bsint; =F. 10. ¢=2-t,y=80; t=1. 15. c= log(t + 2), y=t; t= 2. In the following curves find lengths of (a) subtangent, (b) subnormal, (c) tangent, (@) normal, at any point : Oa { a(cost + tsind), : y = a(sint — t cost). Ans. (a) ycott, (b) y tant, (c) _ @ Pa 4a cost, =4dasint, 17. The hypocyeloid (astroid) { Yy Ans. (a) —ycott, (b) —ytant, (¢) 2, @ 4. (@) ~voott, 0) — tant, (0) 2 (a) 2G (2=rcost, 18. The circle : yersint. =a(2 t— 21) 19. The cardioid te a y =a(2sint—sin20, Bt 148" ). The folium 20. The folium a U Tee a= feos, 21, The hyperbolic spiral y= Ssint t 67. Angle between the radius vector drawn to a point on a curve and the tangent to the curve at that point. Let the equation of the curve in polar codrdinates be p = (8). Let P be any fixed point (p, 8) on the curve. ENO If 6, which we assume as the independent vari- Ip able, takes on an increment Ad, then p will A, take on a corresponding increment Ap. Denote by @ the point (p+Ap, @-+A0). Draw PR perpendicular to 0@. Then 0Q=p+Ap, PR =psin AO, and OR = pcos Ad. Also, PR_ PR psin AO tan POR =F5=Go— Oh p+ Ap—pcosAd 84 DIFFERENTIAL CALCULUS Denote by the angle between the radius vector OP and the tangent PT. If we now let A@ approach the limit zero, then (a) the point Q will approach indefinitely near P; (b) the secant PQ will approach the tangent PT as a limiting posi- tion; and (e) the angle PQR will approach yp as a limit. Hence limit ___psin A@ ti = eee an Y= 49-0 ppap—p cos Ab EY _ limit __psin Ag fu & X [since trom a0, p. 29 eos sep cosa) =2paine 82. psin Ad _ limit Ae ~Ad=0 2psineA?® 2 Ap ae +20 {Dividing both numerator and denominator by 86) _sin AO — limit ary Ag=0 AG. nS psin a + me limit /,, 40) _ limit /sin AQ) _ aoe (sm =0, Vl ocd Ao simi imi : and gp ggg aE bY $.2% pl, we have 2 (A) tan = £. - From the triangle OP’ we get @ rong SIMPLE APPLICATIONS OF THE DERIVATIVE 85, Having found 7, we may then find tan, the slope of the tangent to the curve at P. Or since, from (B), tan 7 = tan (0 +) we may calculate tan y from (A) and substitute in the formula tan@ + tan (C) slope of tangent = tan 7 = Iutusrrative Exanrre 1. Find y and in the eardioid p = a(1—cos6), Also find the slope at @ = 3 : Solution. A = asin@, Substituting in (A) gives = (1 — cos) ons 6 pal = > = tan8. i tany = = ane FH xy By sop. 2, and 37, p.2 7] 2asin 3 cos a8 ve a Since tany=tan5.y=S- Ans, Substituting in (B), r= 6 +3 tan = tan = Ans. To find the angle of intersection ¢ of two curves C and C! whose equations are given in polar codrdinates, we may proceed as follows : angle TPT" = angle OP7'— angle OPT, c or, g=y'—y Hence (D) tang = tn y" = tan y 1+ tany' tay’ where tan yp! and tan y are calculated by (A) from the two curves and evaluated for the point of intersection. Itustrative Exampie 2. Find the angle of x intersection of the curves p = asin26, p = acos20. Solution. Solving the two equations simultaneously, we get at the point of inter section tan26=1, 20 = 45°, 6 = 223°. From the first curve, using (4), tan y/ = } tan 26 = 3, for @ = 224°. ‘From the second curve, tany =— J cot28 Substituting in (D), t 3, for @ = 224°, = are tan $. Ans, 86 DIFFERENTIAL CALCULUS 68. Lengths of polar subtangent and polar subnormal. Draw a line NT through the origin perpendicular to the radius vector of the 7 point P on the curve. If PZ is the tangent and PN the normal to the curve at P, then Lp OF = length of polar subtangent, ol x and ON = length of polar subnormal of the curve at P. A Or In the triangle OP7, tan = —- Therefore Mp Pp (7) OT =ptany=p* 2 = length of polar subtangent.* ip In the triangle OPN, tany =... Therefore 8 on’ p__4p (8) ON=——=+4= . (8) an 6 length of polar subnormal. The length of the polar tangent (= PT’) and the length of the polar normal (= PN) may be found from the figure, each being the hypot- enuse of a right triangle. Intusrrative Examrrx 8. Find lengths of polar subtangent and subnormal to the lemniseate p? = a? cos 26. Solution. Differentiating the equation of the curve as an implicit function with respect to 8, 2p = 2a2sin 20, = dp __atsin2a : a6 o Substituting in (7) and (8), we get length of polar subtangent =~?» _, os . Pain 20 = length of polar snbnormal =— 2226, ° If we wish to express the results in terms of 8, find p in terms of 8 from the given equation and substitute. Thus, in the above, p= +4 aVcos20; therefore length of polar subtangent = + a cot 26-Vcos 20. * When 0 increases with p, 7 fs positive and ¥ is an acute angle, as in the above figure. ‘Then the subtangent 0 T'is positive and is measured to the right of an observer placed at Oand looking along OP. When 7 is negative, the subtangent is negative and is measured to the left of the observer. SIMPLE APPLICATIONS OF THE DERIVATIVE 87 EXAMPLES I. In the circle p = r sind, find y and 7 in terms of 8. Ans. Y=0,7=28. 2. In the parabola p= a sect, show that r+ y=. 3. In the curve p? = a? cos 26, show that 2y = 7 + 46. 4, Show that y is constant in the logarithmic spiral p = e#®, Since the tangent makes a constant angle with the radius vector, this curve is also called the equi- angular spiral. 5. Given the eurve p 4y. asin, prove that 7 &. Show that tan ¥=@ in the spiral of Archimedes p= a9. Find values of y when = 27 and 47. Ans. y = 80° 57’ and 85°27’. 7. Find the angle between the straight line p cos@=2a and the circle p= Sasind. Ans. are tan}. 8, Show that the parabolas p= a sects and p = bese? g intersect at right angles, 9. Find the angle of intersection of p= asing and p= asin 26. Ans. At origin 0°; at two other points are tan8-V3. 10. Find the slopes of the following curves at the points designated : © (a) p= a(1—cosé). 6 Ans, —1. (b) p=asect9. p=2a. ES () p=asin4g, origin. 0, 1,0, —1. (@) p?=atsin 49. origin. 0, Le, —1. (e) p=asin36. origin, 0, V8, — V3. () p= acos8d. origin, (g) p= «@cos26. (h) p=asin29. @) p=asin3. () p=ad (&) p9=a. p=e. 11. Prove that the spiral of Archimedes p = a8, and the reciprocal spiral p=“, intersect at right angles. a 12, Find the angle between the parabola p= a sec? : and the straight line psind=2a. Ans, 45°, 18. Show that the two cardioids p= a(1+ cos) and p=a(1— cos) cut each other perpendicularly. 14. Find lengths of subtangent, subnormal, tangent, and normal of the spiral of : a ; Archimedes p= af. Ans, suit, <2, tan, VET subn, =a, nor. Va? + p*. ‘The student should note the fact that the subnormal is constant. 88 DIFFERENTIAL CALCULUS 15. Get lengths of subtangent, subnormal, tangent, and normal in the logarithmic spiral p= a, Ans. subt.=—°—, tan=pafi¢—, Toga Tog? a subn.=p loga, nor. = p V+ log?a. ‘When a =e, we notice that subt. = subn., and tan. = nor. 16. Find the angles between the curves p = a(1 + cos6), p = b(1— cos6). a Ans. OandZ. 17. Show that the reefprocal spiral p = $ has a constant subtangent. 2 18. Show that the equilateral hyperbolas p® sin 24 = a, p® cos 26 = J? intersect at right angles. 69. Solution of equations having multiple roots. Any root which occurs more than once in an equation is called a multiple root. Thus 38, 3, 3, —2 are the roots of (A) at —Tad +92" + 272-54 = hence 8 is a multiple root occurring three times. Evidently (4) may also be written in the form (e—3)°@+2)=0. Let f(z) denote an integral rational function of x having a multiple root a, and suppose it occurs m times. Then we may write B® F@O=@—O"9@), where $(z) is the product of the factors corresponding to all the roots of f(z) differing from «. Differentiating (B), i. SP@=@- OF @) + b@) mean, Ce) SO=@-O""[E-DP@)+4b@)m]- Therefore f"(z) contains the factor (e—a) repeated m—1 times and no more; that is, the highest common factor (H.C.F.) of f(«) and f(z) has m—1 roots equal to a. In case f(z) has a second multiple root 8 occurring r times, it is evident that the H.C.F, would also contain the factor (e —8)'-}, and so on for any number of different multiple roots, each occurring once more in f(#) than in the H.C.F. We may then state a rule for finding the multiple roots of an equation (2) =0 as follows: First Step. Find f(x). Sroonn Sree. Find the H.C.P. of f(#) and f'(2). Turrp Srer. Find the roots of the H.C.F. Each different root of the B:CLE. will occur once more in f(a) than it dovs in the H.C.F. SIMPLE APPLICATIONS OF THE DERIVATIVE 89 If it turns out that the H.C.F. does not involve 2, then f(2) has no multiple roots and the above process is of no assistance in the solution of the equation, but it may be of interest to know that the equation has no equal, i.e. multiple, roots. Iunustrative Examrie 1, Solve the equation 2? — 82% + 182—6=0. Solution. Place S(t) =a — Ba? 4 132-6, First step. Sa) = 822 — 162418 Second step. HCF,=2-1. Third step. z-1=0. «. Sinee 1 occurs once as a root in the H.C.F., it will oceur twice in the given equa- tion ; that is, (e — 1)? will occur there as a factor. Dividing 2° — 82? + 182 — 6 by (« —1)? gives the only remaining factor (z — 6), yielding the root 6. The roots of our equation are then 1, 1, 6. Drawing the graph of the function, we see that at the double root z= 1 the graph touches OX but does not cross it.* EXAMPLES Solve the first ten equations by the method of this section: 1, a — Ta? 4162-12 Ans. 2,2, 3. 2. at 62%— 82-8 -1,-1,-1,3. 3. at — Ta? + Oa? + 27x — 54 = 0, 8, 8,3, —2. 4, at 52° — 9a? 4 812—108=0. 8, 8,3, —4. 5. xt + 623 + 2? — 242 4+16=0, 1,1,-4, 6. at 949 + 284? — 34 —36=0. 8,8, —1,4. 7. ot — 62% + 1027-8 =0. 2,2, 14-V3. 8, oS at 5a tat48244=0. —1,-1,-1,2,2. 9, 25 — 152° +102? + 60z—72=0. 29 6 8 10, a = Bat — 529 + 182? + 242 +10 = 0. =1,-1,-1,84V—-1L Show that the following four equations have no inultiple (equal) roots: LL, 2 + 92% + 22-48 =0. 12, at — 15a? — 102 + 24 = 0. 18. at — 323 — 6a? + 14a +1: 14. 7 -— a= 15, Show that the condition that the equation + 3qrtr shall have a double root is 44° + 7? = 0. 0 16. Show that the condition that the equation 4 Spz? +r shall have a double root is r(4p% +r) = 0. Since the first derivative vanishes for every multiple root, it follows that the axis of X is tangent to the graph at all points eorre- sponding to multiple roots. If a multiple root occurs an even number of times, the graph will not eross the axis of X at such a point (see figure); if it occurs an odd number of times, the graph will cross. 90 DIFFERENTIAL CALCULUS 70. Applications of the derivative in mechanics. Velocity. Recti- linear motion. Consider the motion of a point P on the straight line AB. Let s be the distance meas- 8 As. . — ps5 ured from some fixed point as A to any position of P, and let ¢ be the corresponding elapsed time. To each value of ¢ corresponds a position of P and therefore a distance (or space) s. Hence s will be a function of ¢, and we may write s=f(t). Now let ¢ take on an increment At; then s takes on an increment As,* and As . (A) Arm the average velocity of P during the time interval At. If P moves with uniform motion, the above ratio will have the same value for every interval of time and is the velocity at any instant. For the general case of any kind of motion, uniform or not, we define the velocity (time rate of change of s) at any instant as the limit of the ratio “ as At approaches the limit zero; that is, — limit As, ™ At=OAt or, ds 9) v=S. 9) a The velocity is the derivative of the distance (= space) with respect to the time. To show that this agrees with the conception we already have of velocity, let us find the velocity of a falling body at the end of two seconds. By experiment it has been found that a body falling freely from rest in a vacuum near the earth’s surface follows approximately the law (BY s=16.18, where s=space fallen in feet, ¢=time in seconds. Apply the Gen eral Rule, p. 29, to (B). * As being the space or distance passed over in the time A¢, SIMPLE APPLICATIONS OF THE DERIVATIVE 91 Fist Svar. s+As—16.1(¢-+At)?=16.10432.2t-At+16.1(A0% Skconn Star, Av = 32.20-At-+16.1 (A? Tarep Sree. * =32.2-4+16.1At= average velocity throughout the time interval At. Placing t= 2, (©) a = 64.4 £161 At =average velocity throughout the time interval At after two seconds of falling. Our notion of velocity tells us ab once that (C) does not give us the actual velocity at the end of two seconds; for even if we take At very small, say zh; or alg of a second, (C) still gives only the average velocity during the corresponding small interval of'time. But what we do mean by the velocity at the end of two seconds is the limit of the average velocity when At diminishes towards zero; that is, the velocity at the end of two seconds is from (C), 64.4 ft. per second. ‘Thus even the everyday notion of velocity which we get from experi- ence involves the idea of a limit, or in our notation _ limit /As\_ ; =O (a3) = 64.4 ft. per second. The above example illustrates well the notion of a limiting value. ‘The student should be impressed with the idea that a limiting value is a definite, fixed value, not something that is only approximated. Observe that it does not make any difference how small 16.1 At may be taken; it is only the limiting value of 64.4 + 16.1 At, when At diminishes towards zero, that is of importance, and that value is exactly 64.4. 71. Component velocities. Curvilinear motion. The codrdinates x and y of a point P moving in the XY-plane are also functions of the time, and the motion may be defined by means of two equations, : z=fOQ, y=bO-" These are the parametric equations of the path (see § 66, p. 79). * The equation of the path in rectangular codrdinates may be found by eliminating ¢ between these equations. 92 DIFFERENTIAL CALCULUS ‘The horizontal component 2, of v* is the velocity along OX of the projection af of P, and is therefore the time rate of change of x. Hence, from (9), p. 90, when s is replaced by 2, we get ax 10) y= 2, (10) eae In the same way we get the vertical com- ponent, or time rate of change of y, x 11) yas. (1) v= Representing the velocity and its components by veetors, we have at once from the figure ‘ Pawty, or, > ds ‘dea? 12) vel= 0 a7 NG) +( giving the magnitude of the velocity at any instant. If + be the angle which the direction of the velocity makes with the axis of X, we have from the figure, using (9), (10), (11), dy ax dy vy dt vee dt v, _ at 13 i wot; aif; Se da sine = 4 aor ; ae a de at dt dt 72. Acceleration. Rectilinear motion. In general, v will be a function of ¢, and we may write vay). Now let ¢ take on an increment At, then v takes on an increment Av, and Av Ma the average acceleration of P during the time interval At. We define the acceleration a at any instant as the limit of the ratio x as At approaches the limit zero; that is, a — limit (Av), At=0\At dv a=-. dt The acceleration is the derivative of the velocity with respect to the time. or, (id) * The direction of » is along the tangent to the path. SIMPLE APPLICATIONS OF THE DERIVATIVE 93 73. Component accelerations. Curvilinear motion. In treatises on Mechanics it is shown that in curvilinear motion the acceleration is not, like the velocity, directed along the tangent, but toward the coneaye side of the path of motion. It may be resolved into a tan- gential component, a, and a normal component, a,, where a, vy ai a, =s° dt R (2 is the radius of curvature. See § 103.) The acceleration may also be resolved into components parallel to the axes of the path of motion. Following the same plan used in § 71 for finding component velocities, we define the component accelerations parallel to OX and OY, (18) ‘dv,\? ‘dv,\* 1 = om 4 ee ° (Gy+ (Ge which gives the magnitude of the acceleration at any instant. (14a) a, Also, EXAMPLES 1. By experiment it has been found that a body falling freely from rest in a vac uum near the earth's surface follows approximately the law 8 = 16.142, where s = space (height) in feet, ¢= time in seconds. Find the velocity and acceleration (a) at any instant; (b) at end of the first second ; (¢) at end of the fifth second. Solution. (4) s=16.12. (a) Differentiating, (B) ae 82.24, or, from (9), » = 82.2¢ft. per sec. Differentiating again, (C) ee 82.2, or, from (14), a = 82.2 ft, per (sec.)?, which tells us that the acceleration of a falling body is constant ; in other words, the velocity increases 82.2 ft, per see. every second it keeps on falling. (b) To find v and a at the end of the first second, substitute t= 1 in (B) and (C) ; v= 82.2 ft, per sec., @ = 32.2 ft, per (sec.)?. (o) Tofind v and a at the end of the fifth second, substitute t= 6 in (B) and (0) ; v= 161 ft. per sec., a = 82.2 ft. per (sec.)?. 2. Neglecting the resistance of the air, the equations of motion for a projectile are r= vcosg-t, y=v, sing-t— 16.12; yy where », = initial velocity, g = angle of projection with hori- zon, t= time of flight in seconds, 2 and y being measured in a feet. Find the velocity, acceleration, component velocities, iA and component accelerations (a) at any instant ; (b) at the end of the first second, having given v, = 100 ft, per sec., ¢ = 30°; (c) find direction of motion at the end of the first second, ax 94 DIFFERENTIAL CALCULUS Solution, From (10) and (11), ® cos; vy = v, sin g — 82.21. ‘Also, from (12), v= Vu? — 64.4 tm, sing + 1086.82. From (15) and (16), ar =0; ay=— 82.2; a=— 82.2. (¥) Substituting = 1, », = 100, g = 30° in these results, we get ng = 86.6 ft. per sec. a= 0. vy = 17.8 ft. per see. ay = — 82.2 ft. per (sec). » = 88.4 ft, per sec. a =— 82.2 ft. per (see.)*. © 7 =aretan™ = are tan "8 — 11° 367.6 = angle of direction of motion with the horizontal, °* a 3. Given the following equations of rectilinear motion, Find the distance, velocity, and acceleration at the instant indicated : Ans. 8=16,0=20,a=16. (b) 6 (c) 8=8— 41; (a) o=2t— {e) y=2t—-B; t= 0. (f) k= 204162; t=10. (g) #=2sint; (h) y= @cos () s=208" (j) 8328 — 36; (ky) e448 () y= oos2t; t= mt =bsin™ 5 ¢ (m) $= sin (n) = ae~2; t (0) sate ue: La ty 4 10 log Saat (p) 8 yt 4, If a projectile be given an initial velocity of 200 ft. per sec. in a direction inclined 45° with the horizontal, find {a) the velocity and direction of motion at the end of the third and sixth seconds; (b) the component velocities at the same instants. Conditions are the same as for Ex. 2. Ans, (a) When t= 8, 148.3 ft, per sec., T= 17°35’, when t= 6, v= 150.5 ft. per sec., t= 159° 53’; (b) when ¢=8, vp = 141.4 ft. per sec., vy = 44.8 ft. per sec. when t = 6, ve = 141.4 ft. per see., vy =— 51.8 ft, per sec, 5, The height (= s) in feet reached in t seconds by a body projected vertically upwards with a velocity of , ft. per see. is given by the formula $= ot — 16.18, SIMPLE APPLICATIONS OF THE DERIVATIVE 95 Find (a) velocity and acceleration at any instant; and, if v,= 800 ft. per sec., find velocity and acceleration (b) at end of 2 seconds; (c) at end of 15 seconds. Resist- ance of air is neglected. Ans, (a) 2= 0, — 82.21, a =— 82.2; (b) 0 = 235.6 ft. per sec. upwards, @ = 82.2 ft. per (sec.)? downwards; (c) v= 188 ft. per sec. downwards, 82.2 ft. per (sec.)? downwards. 6, A cannon ball is fired vertically upwards with a muzzle velocity of 644 ft. per sec, Find (a) its velocity at the end of 10 seconds; (b) for how long it will continue to rise. Conditions same as for Ex, 5, Ans, (a) 322 ft, per sec, upwards; (b) 20 seconds, 7. A train left a station and in ¢ hours was at a distance (space) of P+ 2h +8e : miles from the starting point. Find its acceleration (a) at the end of ¢ hours; (b) at the end of 2 hours, Ans, (a) a= 6t+4; (b) @=16 miles per (hour), 8. In ¢ hours a train had reached a point at the distance of } 4 — 44 4 16 miles from the starting’ point. (a) Find its velocity and acceleration. (b) When will the train stop to change the direction of its motion? (c) Describe the motion during the first 10 hours, Ans. (a) v= 8 —120 4 821, a= 80 ~ 241 + 82; (b) at end of fourth and eighth hours; (c) forward first 4 hours, backward the next 4 hours, forward again after 8 hours. a= 9. The space in feet described in ¢ seconds by a point is expressed by the formula $= 48t—168, Find the velocity and acceleration at the end of 1} seconds. Ans, 0=0,a= 82 ft. per (sec,)®. 10. Find the acceleration, having given ° Ans, « (a) v=acos8t; t=7 (e) v= 58; t= a=We, 11, At the end of t seconds a body has a velocity of 81 4 2t ft. per sec,; find its acceleration (a) in general ; (b) at the end of 4 seconds. Ans. (a) «= 6t+ 2 ft. per (sec,)?; (b) a = 26 ft. per (sec.)? 12, The vertical component of velocity of a point at the end of t seconds is vy = 8 — 20+ 6 ft, per sec, Find the vertical component of acceleration (a) at any instant; (b) at the end of 2 seconds, Ans, (a) @y = 6t—2; (b) 10 ft, per (see.)? 18. If a point moves in a fixed path so that sav, show that the acceleration is negative and proportional to the cube of the velocity. 96 DIFFERENTIAL CALCULUS 14, If the space described is given by aet + be-, show that the acceleration is always equal in magnitude to the space passed over. 15, If a point referred to rectangular codrdinates moves so that © acost +b, and y= asint +c, show that its velocity has a constant magnitude. 16, If the path of a moving point is the sine curve % = at, bsinat, show (a) that the «-component of the velocity is constant; (b) that the acceleration of the point at any instant is proportional to its distance from the axis of X. 17. Given the following equations of curvilinear motion, find at the given instant Tey My, P} Gz, y, ; position of point (codrdinates) ; direction of motion, Also find the equation of the path in rectangular codrdinates, @) e=Ay=t; t=2 (g) ©=2sint, y = Beost; t= m in Seyler aie (h) 2 =sint, y = cos2t; t=3. © 3t=38. i (@ e=2t y=043; t=0, (@) @=2t,y=8e; t=0. © Gj) 2 = 86, y =logt; t=1, ® 2 (k) z=t,y=12t-1; t CHAPTER VII SUCCESSIVE DIFFERENTIATION 74. Definition of successive derivatives. We have seen that the derivative of a function of 2 is in general also a function of 2. This new function may also be differentiable, in which case the derivative of the first derivative is called the second derivative of the original function. Similarly, the derivative of the second derivative is called the third derivative ; and so on to the nth derivative. Thus, if y=Br', W120 a (ay z(2)- 2 A[4 (ay) _ 79 alz (3)|- 722, ete. 75. Notation. The symbols for the successive derivatives are usually abbreviated as follows: d (ay _ ay dc\dz)~ de” If y=f(@), the successive derivatives are also denoted by POs Or FNCOr FOn oy FOC): Peo PP ry YS «, 24), Sra, Sr, Sr@, + Zr. oT 98 DIFFERENTIAL CALCULUS 76. The nth derivative. For certain functions a general expression involving % may be found for the nth derivative. The usual plan is to find a number of the first successive derivatives, as many as may be necessary to discover their law of formation, and then by induction write down the nth derivative. Inuusrrative Examene 1. Given y = e®, find eu. Solution, : ana 2% Tuwusmnative Examene 2, Given y= log, find 5. Solution. ‘ : ay Tuuwsrmative Examrie 8, Given y=sine, find 72, Solution © cosa = sin ¢ + 2) - ot = cos = 7), 2 oe og + z= cos (2 +2) = sin (2 + =), 2 pa Zain | ae a i (e ae %). Ans, 77. Leibnitz's Formula for the nth derivative of a product. This formula expresses the nth derivative of the product of two variables in terms of the variables themselves and their successive derivatives. SUCCESSIVE DIFFERENTIATION 99 If u and » are functions of z, we have, from V, Sw = Boe uk. Differentiating again with respect to 2, du dudv , dudv , d’v_ d*u du dv dv Wee egae ae ae gods aeman ean ae Simin, "nytt, , Pu de , odudr, gdudo , dude, de S dx* dx dx da? dx dx dx? dx dx® dz Mu gdudv, gdudy, dv “wpa aa eS However far this process may be continued, it will be seen that the numerical coefficients follow the same law as those of the Binomial Theorem, and the indices of the derivatives correspond to the expo- nents of the Binomial Theorem.* Reasoning then by mathematical Induction from the mth to the (m+1)th derivative of the product, we can prove Leibnite's Formula a au | Pou de. n(n—t) atu dy 17) — (uv) = —v pa el a rUrmr—~—sssS caneae tae du atv |’ a a ae Tae 3 Intusrranive Exanrie 1, Given y = é loge, find ey by Leibnitz’s Formula. Solution. Let &, and v= loge; dod th fat, hen EA et a--2 @_ 2 a Substituting in (17), we get PY « ertogn 4 3&3 5 2 @(i0 z+e ae 2). a eT ge OS 2, fou ate * To make this correspondence complete, u and v are considered as al and = 100 DIFFERENTIAL CALCULUS Inuusrrative Exampie 2, Given y = 2%, find oe by Leibnita’s Formula. Solution. Let u=2, and v=ew; du dv Dl then eh Ba @u_, @y moh gant Pu @y gan” <5 = wear, au ony an dan = Substituting in (17), we get ay = share™ 4 2nar- Laer + n(n — 1)ar~Feer = ar 2eez [22a + Qnaz + n(n— 1]. = 78. Successive differentiation of implicit functions. To illustrate the : process we shall find oy from the equation of the hyperbola Be aty'= a8. Differentiating with respect to 2, as in § 63, p. 69, dy 282 —2a’y Y= 0, eae or, dy _ bx )) on ay Differentiating again, remembering that y is a function of 2, dy yb? — bina ay ae ae ay Substituting for 4Y its vatue from (4), : aly — ale (=) 2 @y) RCL ay? Sey eae ay _ dat ay ay But from the given equation, b’z*— a°y*= al? dye dat SUCCESSIVE DIFFERENTIATION 101 EXAMPLES Differentiate the following : “Ly = 408 — 622 4 4047. f= 102-0, a wa) = 2 S@= PO) = Tae 3. fy=¥. FAQ) =[8. 4. y= a logz. m8. ¢ _n(nt ie By=s. y= Be 6. y=(@—B)er + dren +2, Sere, 8. f(t) = ax? + be +e. 9. f(c) = log(e +1). 10. f(t) = log (@ + €-*), a r=sinaé, = tang, 13, r= logsin g. 14. f() = e-tcost. 15. (0) =Vs0e 20. 16. p= (72 + @)aretan!. a =ar. = log (1 +2). 19. y = cosac. 20, y =2"-Noge. 21. y= : aie y= 4e[(e— Her +a]. rel @peyat y= sie +e FB rreao iO) =- aa re) =— : em S 4. = = atsin ad = atr, S = 6sectg — 4sec%p. 1" = Deot p esc? F'\() =—4e- "cost =— 4f(). FO) = BF} —F0)- —_e ag (+e) oy = (log ay"ar. oY _ (ay cee ‘Ge | _ _ 2 — wom (+ *2) dry (m= Cr =2(- 194 2 Gp apt Hint. Reduce fraction to form — 14 ed before differentiating. +2 22. If y = eesina, prove that os fy, 23, If y = acos(log2) + bsin (log2), prove that 2¢—¥ 4 2% 4 y 0, ae ae 102 DIFFERENTIAL CALCULUS Use Leibnitz's Formula in the next four examples: 24. y= 2%, fu = a*(loga)"~2{(@ loga +n)? — n] 25. y= ex. Fe exe +m. 26. f(a) = esine. FO) = (V2)reesin ( [ =). 21. (8) = c0s.a8 cosdo. foon(gy = a+ = DY 08 [@ +004 3 +45 Meale-nes2} 28, Show that the formulas for acceleration, (14), (15), p. 92, may be written ey. de 29, y= dace, | 30. bia? + ay? = a0? _ i aye Ble yar, i 32. ty =a 33, ax? + 2hey + by? =1. oe. (ia + te Ca Sate 34. yf? —22y = a — (y—2)8" aes was 35. sec g cos tan? — tantg | tanto = 2(5 +86? + 864) 36. 6 =tan(p +6). -Aeeee. 87. Find the second derivative in the following: (a) log(u +x) =u—v. (e) y+ 2° —3ary=0, (b) + u=erto. (f) Y —2may + 2*9-a=0, (c) s= ltl. (g) y=sin@ +). @ e+st—e=0. (a) ety = ay, CHAPTER VIII MAXIMA AND MINIMA. POINTS OF INFLECTION. CURVE TRACING 79. Introduction. A great many practical problems occur where we have to deal with functions of such a nature that they have a greatest (maximum) value or a least (minimum) value,* and it is very important to know what particular value of the variable gives such a value of the function, For instance, suppose that it is required to find the dimensions of the rectangle of greatest area that can be inscribed in a circle of radius 5 inches. Consider the circle in the following figure : Inseribe any rectangle, as BD. Let CD= 2; then DE =V100 —23, and the area of the rectangle is evidently @ A=xV100—2, ‘That a rectangle of maximum area must exist may be seen as follows: Let the base CD (=2) increase to 10 inches (the diameter); then the altitude D&Z=V100 —a? will decrease to zero and the area will become zero. Now let the base decrease to zero; then the altitude will increase to 10 inches and the area will again become zero. It is therefore intuitionally evident that there exists a greatest rectangle. By a careful study of the figure we might sus- pect that when the rectangle becomes a square its area would be the greatest, but this would at best be mere guess- work. A better way would evidently be to plot the graph of the function (1) and note its behavior. To aid us in drawing the graph of (1), we observe that (a) from the nature of the problem it is evident that 2 and 4 must both be positive; and (b) the values of x range from zero to 10 inclusive. * There may be more than one of each, as illustrated on p. 109. 108 104 DIFFERENTIAL CALCULUS Now construct a table of values and draw the graph. What do we learn from the graph? 19.6 28.6 36.6 43.0 48.0 49.7 48.0 39.6 0.0 Secwrcaawwne (a) If carefully drawn, we may find quite accurately the area of the rectangle corresponding to any value of x by measuring the length of the corresponding ordinate. ‘Thus, when z= 0M=8 inches, then A= MP = 28.6 square inches 5 and when r= ON =4} inches, then A=NQ= about 39.8 sq. in. (found by measurement). (b) There is one horizontal tangent (RS). The ordinate 7H from its point of contact 7’ is greater than any other ordinate. Hence this discovery: One of the inscribed rectangles has evidently a greater area than any of the others. In other words, we may infer from this that the function defined by (1) has a mazimum value, We cannot find this value (= HZ) exactly by measurement, but it is very easy to find, using Calculus methods. We observed that at 7’ the tangent was horizontal; hence the slope will be zero at that point (Illustrative Example 1, p. 74). To find the abscissa of 7 we then find the first derivative of (1), place it equal to zero, and solve for 2. Thus om A=2V100—2, dA _100—227 dz V100—2? 100-22" _ 4 MAXIMA AND MINIMA 105 Solving, x=5V2, Substituting back, we get D#=V100—2 =5V2. Hence the rectangle of maximum area inscribed in the circle is a square of area os A=CD x DE=5V2 x 5V2=50 square inches. The length of HT is therefore 50. ; Take another example. A wooden box is to be built to contain 108 cu. ft. It is to have an open top and a square base. What must be its dimensions in order that the amount of material required shall be a minimum; that is, what dimensions will make the cost the least ? Let =length of side of square base in feet, and _y=height of box. Since the volume of the box is given, how- |i ever, y may be found in terms of 2. Thus volume = 2*y =1083 .-. y=. We may now express the number (=I) of square feet of lumber required as a function of z as follows: area of base = 2? sq, ft., and area of four sides = 4ay = a2 sq. fl. Hence 432 (©) M=274+— z My { | sort mp t x | ar { ort 1 | 438 1 2 | 220 =| 1 3 | 158 sso; 3 [am 2 ri | ' 6 | 108 el 118 7 | a mr | fod ae of | id 1 tot ' ' 10 | 143 ele ' ia oft sa is a formula giving the number of square feet required in any such box having a capacity of 108 cu. ft. Draw a graph of (2). 106 DIFFERENTIAL CALCULUS. What do we learn from the graph? (a) If carefully drawn, we may measure the ordinate correspond- ing to any length (=) of the side of the square base and so deter- mine the number of square feet of lumber required. (b) There is one horizontal tangent (RS). ‘The ordinate from its point of contact 7 is less than any other ordinate. Hence this dis- covery: One of the boxes evidently takes less lumber than any of the others. In other words, we may infer that the function defined by (2) has @ minimum value. Let us find this point on the graph ex- actly, using our Calculus. Differentiating (2) to get the slope at any point, we have 432 ee At the lowest point 7’ the slope will be zero. Hence 432 ee that is, when z= 6 the least amount of lumber will be needed. Substituting in (2), we see that this is M=108 sq. ft. The fact that a least value of If exists is also shown by the follow- ing reasoning. Let the base increase from a very small square to a very large one. In the former case the height must be very great and therefore the amount of lumber required will be large. In the latter case, while the height is small, the base will take a great deal of lumber. Hence M varies from a large value, grows less, then increases again to another large value. It follows, then, that the graph must have a “lowest” point corresponding to the dimensions which require the least amount of lumber, and therefore would involve the least cost. We will now proceed to the treatment in detail of the subject of maxima and minima. 80. Increasing and decreasing functions.* A function is said to be increasing when it increases as the variable increases and decreases as the variable decreases. A function is said to be decreasing when it decreases as the variable increases and increases as the variable decreases. ‘The proofs given here depend chiefly on geometric intuition. The subject of Maxima, and Minima will be treated analytically in § 108, p. 167. MAXIMA AND MINIMA 107 The graph of a function indicates plainly whether it is increasing or decreasing. For instance, consider the function @* whose graph (Fig. a) is the locus of the equation yore. a>1 As we move along the curve from left to right the curve is rising + that is, as 2 increases the function (= y) always increases. ‘Therefore a* is an increasing function for all values of 2. Fre. a Fie. b On the other hand, consider the function (a—2)* whose graph (Fig. 2) is the locus of the equation y=(@—2). Now as we move along the curve from left to right the curve is ‘falling ; that is, as x increases, the function (= y) always decreases. Hence (a—z)* is a decreasing function for all values of 2. : That a function may be sometimes increas- ing and sometimes decreasing is shown by the graph (Fig. c) of y=2a®— Gat +1203. As we move along the curve from left to right the curve rises until we reach the point 4, then it falls from A to B, and to the right of B it is always rising. Hence Fie. (a) from z=— @ tox =1 the function is inereasing ; (b) from x to x= 2 the function is decreasing ; (c) from x= 2 to x=+ 0 the function is increasing. | | 108 DIFFERENTIAL CALCULUS The student should study the curve carefully in order to note the behavior of the function when z=1 and x= 2 Evidently A and B are turning points. At 4 the function ceases to increase and com- mences to decrease; at B, the reverse is true. At A and B the tan- gent (or curve) is evidently parallel to the axis of X, and therefore the slope is zero. 81. Tests for determining when a function is increasing and when decreasing. It is evident from Fig. ¢ that at a point, as C, where a funetion y=f@ is inereasing, the tangent in general makes an acute angle with the axis of X; hence d slope = tant = 22. = f(a) = a positive number. dx Similarly, at a point, as D, where a function is decreasing, the tan- gent in general makes an obtuse angle with the axis of X; therefore slope = tan 7 = Wt = #1(2) = a negative number.* a Jn order, then, that the function shall change from an increasing to a decreasing function, or vice versa, it is a necessary and sufficient condition that the first derivative shall change sign. But this can only happen for a continuous derivative by passing through the value zero. Thus in Fig. ¢, p. 107, as we pass along the curve the derivative (=slope) changes sign at A and B where it has the value zero. In general, then, we have at turning points (18) The derivative is continuous in nearly all our important applica- tions, but it is interesting to note the case when the derivative (= slope) changes sign by passing through «.f This would evidently *Conversely, for any given value of 2, iff’ (e) =4, then f (2) is increasing ; Uf’ (2) =~, then f (x) is decreasing. When “(2) = 0, we cannot decide without further investigation whether f(z) is increas. ing or decreasin, '{ By this is meant that its reciprocal passes through the value zero. MAXIMA AND MINIMA 109 happen at the points B, HZ, G in the following figure, where the tangents (and curve) are perpendicular to the axis of X. At such exceptional turning points YY _ 912) = 03 Pas @=03 or, what amounts to the same thing, A 0 I@) 82, Maximum and minimum values of a function. A maximum value of a function is one that is greater than any values immediately preceding or following. A minimum value of a function is one that is less than any values immediately preceding or following. Fie. d For example, in Fig. c, p. 107, it is clear that the function has a maximum value Jf4 (= y = 2) when «=1, and a minimum value NB (=y =) when x= 2. The student should observe that a maximum value is not neces- sarily the greatest possible value of a function nor a minimum value the least. For in Fig. e it is seen that the function (= y) has values to the right of B that are greater than the maximum JA, and values to the left of A that are less than the minimum VB. A function may have several maximum and minimum values. Suppose that the above figure represents the graph of a function f@)- ‘At B, D, G, I, K the function is a maximum, and at C,H, H, Ja minimum. That some particular minimum value of a function may be greater than some particular maximum value is shown in the figure, the minimum values at C and H being greater than the maximum value at K. 110 DIFFERENTIAL CALCULUS At the ordinary turning points C, D, #, 1 J, K the tangent (or curve) is parallel to OX; therefore slope = & =fl@)=0. ‘At the exceptional turning points B, B, G the tangent (or curve) is perpendicular to OX, giving slope = = (2) =20. One of these two conditions is then necessary in order that the function shall have a maximum ora minimum value. But such a con- dition is not sufficient; for at # the slope is zero and at it is infinite, and yet the function has neither a maximum nor a minimum value at either point. It is necessary for us to know, in addition, how the function behaves in the neighborhood of each point. Thus at the points of maximum value, B,D, G, I, K, the function changes from an increasing to a decreasing function, and at the points of minimum value, C, B, H, J, the function changes from a decreasing to an increasing func- tion. Tt therefore follows from § 81 that at mazimum points slope = # = f"(a) must change from + to—, and at minimum points slope = % = f'(a) must change from — to + when we move along the curve from left to right. ‘At such points as A and F where the slope is zero or infinite, but which are neither mazimum nor minimum points, slope = # = f"(#) does not change sign. We may then state the conditions in general for maximum and minimum values of f (2) for certain values of the variable as follows: (19) f(x) is a maximum if f'(x) = 0, and f!(x) changes from + to—. 20) f(x) is a minimum if f'(~) = 0, and f'(x) changes from — to+. ; ‘The values of the variable at the turning points of a function are called critical values ; thus z=1 and z= 2 are the critical values of MAXIMA AND MINIMA 111 the variable for the function whose graph is shown in Fig. , p. 107. The critical values at turing points where the tangent is parallel to OX are evidently found by placing the first derivative equal to zero and solving for real values of z, just as under § 64, p. 73." To determine the sign of the first derivative at points near a par- ticular turning point, substitute in it, first, a value of the variable just a little less than the corresponding critical value, and then one a little greater.t If the first gives + (as at L, Fig. d, p. 109) and the second — (as at Jf), then the function (= y) has a maximum value in that interval (as at I). If the first gives — (as at P) and the second + (as at 1), then the function (= y) has a minimum value in that interval (as at C). If the sign is the same in both cases (as at Q@ and R), then the function (= y) has neither a maximum nor a minimum value in that interval (as at 7). We shall now summarize our results into a compact working rule. 83. First method for examining a function for maximum and mini- mum values. Working rule. Fimst Srer, Find the first derivative of the function. Ssconp Srer. Set the first derivative equal to zero$ and solve the resulting equation for real roots in order to find the critical values of the variable. Tump Srep. Write the derivative in factor forms. if it is algebraic, write it in linear form. Fourtu Strep. Considering one critical value at a time, test the first derivative, first for a value a trifle less and then for a value a trifle greater than the critical value. If the sign of the derivative is first + and then—, the function has a maximum value for that particular critical value of the variable ; but if the reverse is true, then it has a minimum value. If the sign does not change, the function has neither. * Similarly, if we wish to examine a function at exceptional turning points where the tan- gent is perpendicular to OX, we set the reciprocal of the first derivative equal to zero and solve to find critical values. In this connection the term “little less,” or "trifle less,”” means any value between the next smaller root (critical value) and the one under consideration; and the term “little greater,” or “trifle greater,” means any value between the root under consideration and the next larger one. }A similar discussion will evidently hold for the exceptional turning points B, Z, and 4 respectively. § When the first derivative becomes infinite for a certain value of the independent vari- able, then the function should be examined for such a critical value of the variable, for it may give maximum or minimum values, as at B, E, or A (Fig. d, p. 109). See footnote on p. 108, 112 DIFFERENTIAL CALCULUS In the problem worked out on p. 104 we showed by means of the graph of the function A=xV100—2? that the rectangle of maximum area inscribed in a circle of radius 5 inches contained 50 square inches. ‘This may now be proved ana- lytically as follows by applying the above rule. Solution. F(@) =2Vi00—a. First step. I(ey= is Second step. a oer 0, =a a=5Vv2, which is the critical value. Only the positive sign of the radical is taken, since, from the nature of the problem, the negative sign has no meaning. 2(5-V2—2) (5V2 +2), Third step. res -#) v2 +. mune ° VV (10 — 2) (10 + 2) Fourth step. When 2 <5V2, ti(e) = ZH) = V(4)(+) When «>5-V2, w= 2(-)(+) VOHG) Since the sign of the first derivative changes from + to — at « = 5 V2, the function has a maximum value 1(5-V2) = 6V3-5-V2 = 50, Ans. 84. Second method for examining a function for maximum and mini- mum values. From (19), p. 110, it is clear that in the vicinity of a maximum value of f(2), in passing along the graph from left to right, S'(®) changes from + to 0 to —. Hence f'(2) is a decreasing function, and by § 81 we know that its derivative, ie. the second deriy- ative [=/"(2)] of the function itself, is negative or zero. Similarly, we have, from (20), p. 110, that in the vicinity of a minimum value of f(x) F'(a) changes from — to 0 to +. Hence f’(x) is an increasing function and by § 81 it follows that f"(@) is positive or zero. MAXIMA ‘AND MINIMA 113 The student should observe that f(x) is positive not only-at mini. mum points (as at 4) but also at points such as P. For, as a point passes through P in moving from left to right, d ee slope = tant = a = f'(z) ts an inereasing function. he At such a point the curve is said to be concave upwards. Similarly, f"(2) is negative not only at maximum points (as at B) but also at points such as Q. For, as a point passes through Q, slope = tant = % =f"(2) ta a decreasing function, At such a point the curve is said to be concave downwards.* We may then state the sufficient conditions for maximum and mini- mum values of f(z) for certain values of the variable as follows: 21) f(x) is a maximum if f'(x) = 0 and f"(x) = a negative number. 22) (2) isa minimum if f'(x) =0 and f"(x) =a positive number. Following is the corresponding working rule. Finsr Srev. Find the first derivative of the function. Sxconp Strep. Set the first derivative equal to zero and solve the result- ing equation for real roots in order to find the critical values of the variable. Turep Step, Find the second derivative, Fourtn Srep. Substitute each critical value for the variable in the second derivative. If the result is negative, then the function is a maximum Sor that critical value; if the result is positive, the function is a minimum. When f(x) = 0, or does not exist, the above process fails, although there may even then be a maximum or a minimum; in that case the first method given in the last section still holds, being fundamental. Usually this second method does apply, and when the process of find- ing the second derivative is not too long or tedious, it is generally the shortest method. Let us now apply the above rule to test analytically the function Maa +32 x found in the example worked out on p. 105. * At a point where the curve is concave upwards we sometimes say that the curve has a positive bending, and where it is concave downwards a negative bending. 114 DIFFERENTIAL CALCULUS Solution. feyaate 2. First step. _ Second step. = 6, critical value. Third step. Se) = 24 e. Fourth step. (0) =+. Hence (6) = 108, minimum value. ‘The work of finding maximum and minimum values may frequently be simplified by the aid of the following principles, which follow at once from our discussion of the subject. (a) The maximum and minimum vabies of a continuous function must occur alternately. (b) When eis a positive constant, ¢-f (a) is a maximum or a minimum for such values of x, and such only, as make f (a) a maximum or a minimum. Hence, in determining the critical values of # and testing for max- ima and minima, any constant factor may be omitted. When e is negative, ¢-f(a) is a maximum when f(a) is a minimum, and conversely. (©) Ff ¢ is a constant, f(@) and e+ f(a) have maximum and minimum values for the same values of x. Hence a constant term may be omitted when finding critical values of @ and testing, In general we must first construct, from the conditions given in the problem, the function whose maximum and minimum values are required, as was done in the two examples worked out on pp. 103- 106. This is sometimes a problem of considerable difficulty. No rule applicable in all cases can be given for constructing the function, but in a large number of problems we may be guided by the following General directions. (a) Express the function whose maximum or minimum is involved in the problem. (b) If the resulting expression contains more than one variable, the conditions of the problem will furnish enough relations between the varia- bles so that all may be expressed in terms of a single one. MAXIMA AND MINIMA 115 (©) To the resulting function of a single variable apply one of our two rules for finding maximum and minimum values. (A) In practical problems it is usually easy to tell which critical value will give a maximum and which a minimum value, 80 it is not always necessary to apply the fourth step of our rules, (c) Draw the graph of the function (p. 104) in order to check the work, PROBLEMS 1. It is desired to make an open-top box of greatest, possible volume from asquare piece of tin whose side is a, by cutting equal squares out of the corners and then fold- ing up the tin to form the sides, What should be the length of a side of the squares cut out? Solution. Let then a2 side of small square = depth of box ; = side of square forming bottom of box, and volume is, V=(a—22)0; which is the function to be made a maximum by varying z. Applying rule, : First step. ve (—22)? — 4 (a — 22) = a? — Bax 4 120%, Second step. Solving a? — 8 ax + 12.2? = O gives critical values 2 = Sands It is evident from the figure that 2 = A must give a minimum, for then all the tin would be cut away, leaving no material out of which to make a box. By the usual 2 Gis found to give @ maximum volume “ test, + Hence the side of the square to be cut out is one sixth of the side of the given square. The drawing of the graph of the function in this and the following problems is left to the student. 2, Assuming that the strength of a beam with rectangular eross section varies directly as the breadth and as the square of the depth, what are the dimensions of the strongest beam that can be sawed out of a round log whose diameter is d? Solution, If z= breadth and y = depth, then the beam will have maximum strength when the function zy? is a maximum. From the figure, y? = d— 2?; hence we should test the function S(e) = a(@— 24). First step. f(t) =— 202 4 @ — 2? = 32%, Second step. d? — 82° E = critical value which gives a maximum, v3 ‘Therefore, if the beam is cut so that depth = -V¥ of diameter of log, and breadth = Vj of diameter of log, the beam will have maximum strength. , 116 DIFFERENTIAL CALCULUS 3. What is the width of the rectangle of maximum area that can be inscribed in a given segment 04.4’ of a parabola? Huxt. If OC=h, B= h~a and PP’=2y; therefore the area of rectangle PDD'P” is 2h=ay. But since P lies on the parabola y?=2pz, the funetion to be tested is 2(h—-2) V2pe. Z 7 Tj Z. V Ld Ans, Width = jh. 4, Find the altitude of the cone of maximum volume that can be inscribed in a sphere of radius r. B Hrvr, Volume of cone=$2%y. But2?= BCx CD=y (2r-y); there- fore the function to be tested is SW) PCr). ) ——— Ans, Altitude of cone = ¢r. Sa 5, Find the altitude of the cylinder of maximum volume that can be inscribed in a given right cone, Hint. Let AC=rand BC=h. Volume of cylinder= rey, But from similar triangles ABC and DB@ nh), A nm hihn=y. Hence the function to be tested is 2 LO=Fy a v Ans. Altitude = fh. 6, Divide a into two parts such that their product is a maximum, Ans. Each part 2 7. Divide 10 into two such parts that the sum of the double of one and square of ‘the other may be 2 minimum. Ans. 9 and 1. 8. Find the number that exceeds its square by the greatest possible quantity. Ans. 3. 9, What number added to its reciprocal gives the least possible sum? Ans. 1. 10, Assuming that the stiffness of a beam of rectangular cross section varies directly as the breadth and the cube of the depth, what must be the breadth of the stiffest beam that can be cut from a log 16 inches in diameter ? Ans. Breadth = 8 inches, 11. A water tank is to be constrneted with a square base and open top, and is to hold 64 cubic yards, If the cost of the sides is $1 a square yard, and of the bottom $2 a square yard, what are the dimensions when the cost is a minimum ? What is the minimum. cost ? Ans. Side of base = 4 yd., height = 4 yd., cost $06. 12, A rectangular tract of land is to be bought for the purpose of laying out a quarter-mile track with straightaway sides and semicircular ends. In addition a strip 85 yards wide along each straightaway is to be bought for grand atands, training quarters, ete. If the land costs $200 an acre, what will be the maximum cost of the land required ? Ans. $856. MAXIMA AND MINIMA Wt 18. A torpedo boat is anchored 9 miles from the nearest point of a beach, and it is desired to send a messenger in the shortest possible time to a military camp situated 15 miles from that point along the shore. If he cay walk 5 miles an hour but row only 4 miles an hour, required the place he must land. Ans, 3 miles from the camp. 14, A gas holder is a cylindrical vessel closed at the top and open at the bottom, where it sinks into the water. What should be its proportions for a given volume to require the least material (this would also give least weight) Ans. Diameter = double the height. 15, What should be the dimensions and weight of a gas holder of 8,000,000 cubie feet capacity, built in the most economical manner out of sheet iron of an inch thick and weighing 2} Ib, per sq. ft.? Ans. Height = 137 ft., diameter = 278 ft., weight = 220 tons. 16. Asheet of paper is to contain 18 sq. in. of printed matter, The margins at the top and bottom are to be 2 inches each and at the sides 1 inch each, Determine the dimen- sions of the sheet which will require the least amount of paper, Ans. 5 in, by 10 in, 17. A paper-box manufacturer has in stock a quantity of strawboard 30 inches by inches, Out of this material he wishes to make open-top boxes by entting equal squares out of each corner and then folding up to form the sides, Find the side of the square that should be cut out in order to give the boxes maximum volume. Ans. 3 inches. 18. A roofer wishes to make an open gutter of maximum capacity whose bottom and sides are each 4 inches wide and whose sides have the same slope. What should be the width across the top ? Ans. 8 inches, 4 19. Assuming that the energy expended in driving a steamboat through the water varies as the cube of her velocity, find her most economical rate per hour when steam- ing against a current running ¢ miles per hour. Hint. Let v= most economical speed ; then av? = energy expended each hour, a being a constant depending upon the partic- ular conditions, = actual distance advanced per hour. and ve 2 Hence a is the energy expended per mile of distance advanced, and it is therefore the funetion whose minimum is wanted. Ans. v= $e. 20. Prove that a conical tent of a given capacity will require the least amount of canvas when the height is -V2 times the radius of the base, Show that when the canvas is laid out flat it will be a circle with a sector of 152° 9 cut out. A bell tent 10 ft. high should then have a base of diameter 14 ft. and would require 272 sq. ft. of canvas. 21. A cylindrical steam boiler is to be constructed having a capacity of 1000 cu. ft. ‘The material for the side costs $2 a square foot, and for the ends $8 a syuare foot, Find radius when the cost is the least. 10 Ans. —— ft. Var 22, In the corner of a field bounded by two perpendicular roads a spring is situated 6 rods from one road and 8 rods from the other. How should a straight road be ran by this spring and across the corner so as to cut off as little of the field ae possible ? Ans. 12 and 16 rods from corner. ‘What would be the length of the shortest road that could be run across? Ans. (6i 4 8h! rods, 118 DIFFERENTIAL, CALCULUS, 23, Show that a square is the rectangle of maximum perimeter that can be inscribed in a given circle, : 24, Two poles of height a and b féet are standing upright and are c feet apart. Find the point on the line joining their bases such that the sum of the squares of the distances from this point to the tops of the poles isa minimum, Ans, Midway between the poles. When will the sui of these distances be a minimum ? 25. A conical tank with open top is to be built to contain V cubic feet. Determine the shape if the material used is a minimum, 26. An isosceles triangle has a base 12 in, long and altitude 10 in, Find the ree- tangle of maximum area that can be inscribed in it, one side of the rectangle coineid- ing with the base of the triangle. 27. Divide the number 4 into two such parts that the sum of the cube of one part and three times the square of the other shall have a maximuin value, 28. Divide the number a into two parts such that the product of one part by the fourth power of the other part shall be a maxtinum, 29. A can buoy in the form of a double cone fs to be made from two equal cireular iron plates of radius r, Find the radius of the base of the cone when the buoy has the greatest displacement (maximum volume). Ans. rV¥. 80, Into a full conical wineglass of depth a and generating angle a there is care- fully dropped a sphere of such size as to cause the greatest overflow. Show that the radius of the sphere is cane sina-+ cos2a" 31. A wall 27 ft. high is8 ft. from ahouse, Find the length of the shortest ladder that, will reach the house if one end rests on the ground outside of the wall, Ans, 18-V13, 82, A vessel is anchored 3 miles offshore, and opposite a point 5 miles further along the shore another vessel is anchored 9 miles from the shore. A boat from the first vessel is to land a passenger on the shore and then proceed to the other vessel. What is the shortest course of the boat ? Ans, 18 miles, 33, A steel girder 26 ft. long is moved on rollers along a passageway 12.8 ft. wide and into a corridor at right angles to the passageway. Neglecting the width of the girder, how wide must the corridor be? Ans, 5.4 ft, 34, A miner wishes to dig a tunnel from a point A to a point B 800 feet below and 600 feet to the east of A. Below the level of A it is bed rock and above A is soft earth, If the cost of tunneling through earth is $1 and through rock $3 per linear foot, find the minimum cost of a tunnel. Ans, $1348.58. 35. A carpenter has 108 sq. ft. of lumber with which to build a box with a square base and open top. Find the dimensions of the largest possible box he can make, Ans, 6x 6x 8, 36. Find the right triangle of maximum area that can be constructed on a line of length 4 as hypotenuse, h Ans. length of both legs. v2 37. What is the isosceles triangle of maximum area that can be inscribed in a given circle ? Ans. An equilateral triangle, 38, Find the altitude ofthe maximum rectangle that can be inscribed in a right triaugle with base b and altitude h, h Ans, Altitud MAXIMA AND MINIMA 119 39. Find the dimensions of the rectangle of maximum area that can be inscribed in the ellipse ba? + a%y? = ab. Ans. aV2 andb V2; area = 2ab. 40. Find the altitude of the right cylinder of maximum volume that can be inscribed in a sphere of radius r. 2r Ans. Altitude of cylinder ==". V3 3 41, Find the altitude of the right cylinder of maximum convex (curved) surface that can be inscribed in a given sphere. Ans. Altitude of cylinder = r-V2. 42, What are the dimensions of the right hexagonal prism of minimum surface whose volume is 86 cubic feet ? Ans. Altitude = 2-V3; side of hexagon = 2. 43, Find the altitude of the right cone of minimum volume circumscribed about a given sphere. Ans. Altitude = 47, and volume = 2 x vol. of sphere. 44. A right cone of maximum volume is inscribed in a given right cone, the vertex of the inside cone being at the center of the base of the given cone, Show that the altitude of the inside cone is one third the altitude of the given cone. 45. Given a point on the axis of the parabola y? = 2 pz at a distance a from the vertex ; find the abscissa of the point of the curve nearest to it. Ans. @=a—p. 46. What is the length of the shortest line that can be drawn tangent to the ellipse 2? + ay? = ab? and meeting the codrdinate axes? Ans. a+b. 47. A Norman window consists of a rectangle surmounted by a semicircle. Given the perimeter, required. the height and breadth of the window when the quantity of light admitted is a maxtinum. Ans. Radius of circle = height of rectangle. 48. A tapestry 7 feet in height is hung on a wall so that its lower edge is 9 feet above an observer's eye. At what distance from the wail should he stand in order to obtain the most favorable view ? Ans. 12 feet. Hixr. The vertical angle subtended hy the tapestry in the eye of the observer must he ata maximum, 49, What are the most economical proportions of a tin ean which shall have a given capacity, making allowance for waste ? 2Vv38 Ans, Height = 23 x diameter of base. 7 Hivr. There is no waste in eutting ont tin for the side of the can, but for top and bottom a hexagon of tin circumscribing the eireular pieces required is used up. Nore 1, If no allowance is made for waste, then height = diamete Nore 2. We know that the shape of a hee cell is hexagonal, giving a certain capacity for honey with the greatest possible economy of wax. 50. An open cylindrical trough is constructed by bending a given sheet of tin of breadth 2a, Find the radius of the cylinder of which the trough forms a part when the capacity of the trough is a maximum, 2 fae saa Ans, Rad, = ="; i.e, it must be bent in the form of a semicircle 51. A weight Wis to be raised by means of a lever with the force F at one end and the point of support at the other, If the weight is suspended from a point at a distance a from the point of support, and the weight of the beam is w pounds per linear foot, what should be the length of the lever in order that the force required to lift it shall be a minimum ? : Ans. ©=4 ; w 120 DIFFERENTIAL CALCULUS 52. An electric are light is to be placed directly over the center of a cfroular plot of grass 100 feet in diameter. Assuming that the intensity of light varies directly as the sine of the angle under which it strikes an illuminated surface, and inversely as the square of its distance from the surface, how high should the light be hung in order that the best possible light shall fall on a walk along the circumference of the plot? 7 oe ee Va \ 83, The lower corner of a leaf, whose width is a, is folded over so as just to reach the inner edge of the page. (a) Find the width of the part folded over when the length of the crease is a minimum, (b) Find the width when the area folded over isa minimum, Ans. (a) $a; (b) $a. 54, A rectangular stockade is to be built which must have a certain area. If a stone wall already constructed is available for one of the sides, find the dimensions which would make the cost of construction the least. Ans. Side parallel to wall = twice the length of each end. 5B. A cow is tethered by a perfectly smooth rope, a slip noose in the rope being thrown over a large square post, If the cow pulls the rope taut in the direction shown in the figure, at what angle will the rope leave the post ? Ans, 30°, 56, When the resistance of air is taken into account, the inelination of a pendulum to the vertical may be given by the formula, 0 = ae-H cos (nt +). Show that the greatest elongations occur at equal intervals = of time. 57. It is required to measure a certain unknown magnitude « with precision, Suppose that n equally careful observations of the magnitude are made, giving the results a a rr ‘The errors of these observations are evidently B— dy Bay Z—ady + some of which ave positive and some negative, It has been agreed that the most probable value of « is such that it renders the sum of the squares of the errors, namely (= a)? + = a3)? + (@— a)? $0 + (= )?, a minimum. Show that this gives the arithmetical mean of the observations as the most probable value of x. B 58, ‘The bending moment at B of a beam of length 1, uniformly loaded, is given by the formula M=4}ule— }wz?, where w = load per unit length. Show that the maximum bending moment is at the center of the beam, 59. If the total waste per mile in an electric conductor is 2 waertt, [c = constant] Where c= current in amperes, = resistance in ohms per mile, and ¢ =a constant depending on the interest on the investment and the depreciation of the plant, what is the relation between c, r, and t when the waste is a minimum ? Ans. cr=t. MAXIMA AND MINIMA 121 60. A submarine telegraph eable consists of a core of copper wires with a covering made of nonconducting material. If z denote the ratio of the radius of the core to the thickness of the covering, it is known that the speed of signaling varies as a) 7 1 Show that the greatest speed is attained when 2 = 7 ‘e 61. Assuming that the power given out by a voltaic cell is given by the formula ER P=——, w+ where E = constant electromotive force, r= constant inter nal resistance, prove that P is a maximum when r= R. al resistance, 2 = exter- 62. The force exerted by a circular electric current of radius @ on a small magnet whose axis coincides with the axis of the circle varies as. x (@+ah where « = distance of magnet from plane of circle. Prove that the force is a maxi- a mum when 2 = 5- 63. We have two sources of heat at A and B with intensities a and b respectively. ‘The total intensity of heat at a distance of z from A is given by the formula cD 2 Z ¢ x)? : Show that the temperature at P will be the lowest when d-2 % @ that is, the distances BP and AP have the same ratio as the cube roots of the corre- sponding heat intensities. The distance of P from A is ata at ob 64. The range OX of a projectile in a vacuun is given by the formula v2sin2¢. =o initial velocity, g= acceleration due to grav- ity, @ = angle of projection with the horizontal. Find the angle of projection which gives the greatest range for a given initial velocity. Ans. $= 45°. a 65. The total time of flight of the projectile in the last problem is given by the formula Qu, sing gy T At what angle should it be projected in order to make the time of flight a maximum ? Ans. $= 90 122 DIFFERENTIAL CALCULUS 66. The time it takes a ball to roll down an inclined plane AB is given by the fortaula, _— a a yA. T=24/—*_. geindg Neglecting friction, etc., what must be the value of @ to make the quickest descent ? Ans. ¢ = 45°. Ife 67; Examine the function (¢—1)*(¢ +1) for maximum and minimum values, Use the first method, p. 111. Solution. f(e) = (@ —1)2(@ +1). First step. 7'(@) = 2(e —1)(e +1)? +3(@—1)2(¢ +1)? =(@— Ne + 1)%(52—1). Second step. (t —1) (e + 1)2(5¢—1) =0, w=1, —1, }, which are critical values. Third step. F’'@) = 5(@—-1)(@+1%@-}). Fourth step. Examine first for critical value = 1(C in FE) hen 2 <1, Se) = 5(-) (HCE When 2>1, (2) = 6(4)(4)2(4) = +. ‘Therefore, when # = 1 the function has a minimum value f(1)= 0 (= ordinate of C). Examine now for the critical value « = } (B in figure). When 2 <4, (2) = 5(—)(+)*(-) = +. When 2 > 4, f’(e) = 5(—)(+)2(+) = —- ‘Therefore, when # = 4 the function has a maximum value f(}) = 1.11 (= ordinate of B). Examine lastly for the critical value « =— 1 (A in figure). When 2<—1, f(a) = 5(—)(—)*(— When 2 > —1, f’(e) = 5(—) (+)?(—| ‘Therefore, when z =— 1 the function has neither a maximum nor a minimum value. 68. Examine the function a — b(¢— c)# for maxima and minima, Solution. F(a) = a—b@— oft, 2b I'@)=-——. B(e— ot Since z= is a critical value for which /”(z) =e, but for which f (2) is not infinite, let us test the function for maximum and minimum values when z= c. When ao, f’(2) Hence, when z = ¢ = OM the function has a maximum value £(¢) ae Examine the following functions for maximum and minimum values: 69. (« — 8)? (@— 2). Ans, « =}, gives max. = sy; 2 = 8, gives min. = 0. 10 (@—1)(@— 2% w= §, gives max. = .08456; x = 2, gives min, x= 1, gives neithe MAXIMA AND MINIMA 123 TL. (e— 4) (e + 2. Ans, 2 =— 2, gives max.; 2 = 3, gives min.; 2 =4, gives neither, 72. (e — 28 (2a + 1). 2=— }, gives max.; 2 = 4), gives min; 2 =2, gives neither. 73. (2 +)i@— 5). a =}, gives max.; 2=—1and 5, give min, 74, Qe —wte— ah. “ gives max.; =a, gives min; “ » gives neither, 75. e(e— 1)? +1) 4, gives max.; Land — 4, give min.; © =—1, gives neither. 76, x(a +2)?(4 —2)*. —aand P give max.; oc 7 = 5» gives min; a, gives neither, 77. V4 e(e— aye a, gives min, 78, a—b(e—o)*. ‘No max, or min, 2 6 79. eatete. 2 = 4, gives max.; . x =16, gives min. ca £, gives'min. so, f= 78 1—-2+2 i 31, ——_——. a = }, gives min. 8. Se i gi * 82. ae 2 = V3, gives min, = 12-V3—17: e480 : : a =—V2, givesmax, =—12-V2—17; a=—1, — 2, give neither, 83. Goats ead gives max. = a. 2 a OB “, gives min. a a 2=—“_, gives max. att 124 DIFFERENTIAL CALCULUS 85, Examine 2? — 82% — 92 + 5 for ma: a and minima, Use the second method, p. 113. Solution. f(t) =x Bat 945, First step. S'(@) = 822 — ba —9. Second step. 32? —62—9=0; hence the critical valuesare = =— Land 3. Third step. (a) = 62-6. Fourth step. f’(—1) =— 12. +. f(—1) = 10 = (ordinate of A) = maximum value, 78) =+ 12. . f(8) =— 22 (ordinate of B) inimum value, 86, Examine sin?e cose for maximum and minimum values. Solution. Sf (2) = sin?a cosa. First step. f’(e) = 2sinz cost — Second step. 2 sina costa — sin? = 0; Wa, hence the critical values are cour and =n + arctan V2=n7 ba. Third step. F(a) = cos (2 cosa — 7 sin®z), Fourth step. (0) = +. ~. f(0) = 0 = minimum value at 0. ©. £(#) = 0 = maximum value at C. &. F(a) = maximum value at A. f(r — a) = minimum value at B, ete, Examine the following functions for maximum and minimum values. 81. 328 — 92? — 272 + 80, Ans, © =—1, gives max. = 45; 2 = 8, gives min, =— 51. 88, 22° — 2127 + 362 — 20. a =1, gives max.=— 38; 2 =6, gives min, =— 128, 8 89, Froese th 2 =1, gives max. 2 8, gives min, 90. 22° — 152% + 86x +10. = 2, gives max, = 88; = 8, gives min. = 37, 91. a — 9a? + 152-3. , gives max. = 4; 2 = 5, gives mi 28, 92, a? — 82? + 62 +10, No max, or min, 93, 26 — Sat 4 52% 41. 2 =1, gives max, z= 8, gives min, x = 0, gives neither. 94. 325 — 1252% 4 21602, 2 =—4and 8, give max.; a2=—3and 4, give min. 95, 228 — 8at— 12044, 98. at — 4, 96. 22* — 212? + 862 — 20. 99, 2 — 8, 97, at — 22% +10. 100. 4— 2°, 101, 102, 103. 104. 105. 106, 107. 108, 109, 110. mm. 112. 13. 14, 15. 85. called MAXIMA sinz (1+ cosz). loge log cos. ack= + be Bs, sin2z—z. z+ tang. sin? cosa. x eosa. sing + cos2a. 2tang — tanta. sing T+ tang 2 1+atang Ans. 2a 2nm +7, give max. AND MINIMA 125 ™ a= 2nm—F, give min, z= ne, give neither, =e, gives min, =e; z= 1, gives neither. x= nm, gives max, i log e gives min. = 2-Vab. z is gives min. a =e, gives max. a= D gives max. = V2; 2 =, gives min. =— V2, = » gives max,; 6 mo 2=—F: gives min, No max. or min. © 3 ganr +, gives ==V3; +E, gives max. = 5 = nm —F, gives min, = nm, gives neither. z= cota, gives max. are sin}, gives max,; = ‘i a= > gives min, = 7? gives max, ™, gives max. a8 . cosz, gives max. — cos, gives min. Points of inflection. Definition. Points of inflection separate arcs concave upwards from ares concave downwards.* Thus, if a curve y=J(@) changes (as at B) from concave upwards (as at A) to con- cave downwards (as at C’), or the reverse, then such a point as B is a point of inflection. * Points of inflection may also be defined as points where or ay ®y : 2H gana 2 on @) F¥-o.and SY changes sign, ) Fen 0 and 7m changes sign. 126 DIFFERENTIAL CALCULUS From the discussion of § 84 it follows at once that at 4, f"(r)=+, and at C,f"(@)=—. In order to change sign it must pass through the value zero;* hence we have 3) at points of inflection, f(x) = 0. Solving the equation resulting from (28) gives the abscissas of the points of inflection. To determine the direction of curving or direc- tion of bending in the vicinity of a point of in- @ flection, test f"() for values of 2, first a trifle less and then a trifle greater than the abscissa at that point. Iff"(2) changes sign, we have a point of in- flection, and the signs obtained determine if the curve is concave upwards or concave downwards in the neighborhood of each point of inflection. ‘The student should observe that near a point where the curve is concave upwards (as at 4) the curve lies above the tangent, and at a point where the curve is concave downwards (as at ()) the curve lies below the tangent. At a point of inflection (as at B) the tangent: evidently crosses the curve. Following is a rule for finding points of inflection of the curve whose equation is y=/(2). This rule includes also directions for examining the direction of curvature of the curve in the neighborhood of each point of inflection. y| x ¥Fmsr Srer. Find f"(2). Srconp Sruv. Set f"(2) =0, and solve the resulting equation for real roots. Tump Srer. Write f(a) in factor form. Fourrn Srep. Test f"(2) for values of 2, first a trifle less and then'a trifle greater than each root found in the second step. If f!(x) changes sign, we have a point of inflection. When f(x) =+, the curve is concave upwards \+7.1 When fll(x)=—, the curve is eoncave downwards —~. * It is assumed that /’(z) and f”() are continuous. The solution of Ex. 2, p. 127, shows how to discnss a case where f(x) and f’”(z) are both infinite. Evidently salient points (see p. 258) are excluded, since at such points /”(2) is discontinuous. + This may be easily remembered if we say that a vessel shaped like the curve where it is concave upwards will hold (+) water, and where it is concave downwards will spill (-) water.

You might also like